Med Surg Exam 4 - Endocrine System, Med Surg Oncology, Diabetes, HIV and AIDS, Diabetes, Endocrine

Réussis tes devoirs et examens dès maintenant avec Quizwiz!

pathophysiology of lung cancer

- repeated exposure to carcinogen causes epithelial cells to mutate over time to become malignant - molecular changes occur over several decades - grows slowly into large masses which can obstruct bronchi and impairs gas exchange - metastasizes to liver, brain, bone, lymph nodes, and adrenal glands via blood and lymph nodes

late manifestations of colorectal cancer

- sever pain - anorexia - weight loss - sacral/sciatic pain - jaundice - ascites - pruritus - hepatomegaly

colorectal cancer

- slow growing - 5-15 years before symptoms start - may not have symptoms until metastasis - most commonly seeks help for colorectal bleeding

classifications of lung cancer

- small cell - non-small cell carcinoma

risk factors of lung cancer

- smoking - occupational and environmental exposures - genetic susceptibility - inflammatory respiratory disease - diet and nutrition - lack of exercise

management of colorectal cancer

- surgery: primary treatment - removal of cancer and surrounding colon - radiation - chemotherapy - targeted therapy - immunotherapy

treatment of pleural effusion

- thoracentesis - chest tube insertion - pleurodesis

low anterior resection

- type of surgery for rectal cancer - removal of a portion of the distal large intestine - occurs if sphincter preservation is possible

abdominoperineal resection

- type of surgery for rectal cancer - removal of the distal colon and rectum through both abdominal and perineal approaches - resection if sphincter preservation is not possible - results in permanent colonostomy

VATS

- video-assisted thoracic surgery - minimally invasive

Fecal immunochemical test (FIT)

- yearly - direct examination of stool

fecal occult blood testing

- yearly - direct examination of stool

Causes of hypoglycemia

-Alcohol intake without food -Too little food - delayed, omitted, inadequate intake -Too much diabetic medication -Too much exercise without adequate food intake -Diabetes medication or food taken at wrong time -Loss of weight without change in medication -Use of B adrenergic blockers interfering with recognition of symptoms

Manifestations of hypoglycemia

-Blood glucose <70 mg/dL -Cold, clammy skin -Numbness of fingers, toes, and mouth -Rapid heart rate -Emotional changes -Headache -Nervousness, tremors -Faintness, dizziness -Unsteady gait, slurred speech -Hunger -Changes in vision -Seizures, coma

Manifestations of hyperglycemia

-Elevated blood glucose -Increase urination -Increase in appetite followed by lack of appetite -Weakness, fatigue -Blurred vision -Headache -Glycosuria -N/V -Abdominal cramps -Progression to DKA or HHS

Treatment of hyperglycemia

-Get medical care -Continue diabetes medication as ordered -Check blood glucose frequently and check urine for ketones; record results -Drink fluids at least on an hourly basis -Contact health care provider regarding ketonuria

Causes of hyperglycemia

-Illness, infection -Corticosteroids -Too much food -Too little or no diabetes medication -Inactivity -Emotional, physical stress -Poor absorption of insulin

Treatment of hypoglycemia

-Immediately ingest 15 grams of simple carbohydrates -Wait 15 min. Then check blood glucose again. -If blood glucose is <70 mg/dL, repeat 15 g of carbohydrates -Contact emergency medical service (EMS) if no relief obtained -Discuss medication dosage with health care provider

Nursing care r/t insulin therapy

-Proper administration -Assessment of patient's response to insulin therapy -Teaching the patient how to administer, adjust, and the side effects of insulin therapy. This includes the S/S of hypo and hyper glycemia. -They must be able to inject themselves, if they can't something else needs to be arranged. -Follow up assessment including inspection for lipodystrohy (atrophy of subQ tissue) and other reactions -A review of the patient's record of urine and blood glucose tests is also used to overall glycemic control

Preventive measures for hyperglycemia

-Take perscribed dose of medication at proper time -Accurately administer insulin, noninsulin injectables, OA -Maintain diet -Adhere to sick-day rules when ill -Check blood for glucose as ordered -Wear diabetic identification

Onset of Short acting insulins

30-60 minutes

The nurse is to give Lamivudine (3TC) 300 mg PO now and has available a 10 mg/mL oral solution. How many milliliter(s) should the nurse give?

30mL (300÷10) x 1

insulin is a hormone secreted by the Islets of Langerhans and is essential for the metabolism of carbohydrates, fats, and protein. The nurse understands the physiologic importance of gluconeogenesis, which refers to the: A. Storage of glucose as glycogen in the liver. B.Transport of potassium. C. Synthesis of glucose from noncarbohydrate sources. D. Release of glucose.

C

Which is the best nursing explanation for the symptom of polyuria in a client with diabetes mellitus? A. With diabetes, drinking more results in more urine production. B Increased ketones in the urine promote the manufacturing of more urine. C High sugar pulls fluid into the bloodstream, which results in more urine production. D The body's requirement for fuel drives the production of urine.

C

A patient who is being tested for syndrome of inappropriate antidiuretic hormone (SIADH) secretion asks the nurse to explain the diagnosis. The nurse explains that there is an excessive secretion of antidiuretic hormone (ADH) from which of the following glands? a) Thyroid b) Anterior pituitary c) Posterior pituitary d) Adrenal

C. Antidiuretic hormone is secreted by the posterior pituitary gland.

A patient is concerned about telling others he has HIV infection. In discussing his concerns, which response by the nurse is most appropriate? A.There is no reason to hide his disease. B. His diagnosis will be obvious to most people with whom he will come into contact. C. Care providers and sexual partners should be told about his diagnosis. D. Secrecy is a poor idea because it will lower his self-esteem.

C. Care providers and sexual partners should be told about his diagnosis.

The nurse explains that the painful shingles experienced by the patient with HIV is related to his childhood exposure to: A. Mumps B. Measles C. Chickenpox D. Impetigo

C. Chickenpox

The patient, age 21, has been treated for chlamydia and has a history of recurrent herpes. What should the nurse plan to do next? A. Refer this patient to a family planning clinic. B. Counsel this patient about testing for HIV and what the test results mean. C. Counsel this patient about her sexual and drug-use history, risk reduction measures, and testing for HIV. D. Counsel this patient about abstinence and a monogamous relationship.

C. Counsel this patient about her sexual and drug-use history, risk reduction measures, and testing for HIV.

The nurse includes in the teaching plan for a patient with HIV who has been diagnosed with microsporidiosis that the patient should: A. Include milk products with every meal. B. Limit protein intake to reduce serum ammonia levels. C. Drink 3 quarts of fluid a day to combat dehydration. D. Consume liberal amounts of fat for increased energy.

C. Drink 3 quarts of fluid a day to combat dehydration.

Duration of Rapid acting insulins

3-5 hours

Peak of Rapid acting insulins

30 minutes - 180 minutes

small cell carcinoma (SCLC)

- rapid cell growth and centrally located - most malignant and virulent with early metastases - associated with paraneoplastic syndromes - most strongly associated with smoking

mucosal layer

where does colon cancer start?

liver

where is a place of common metastasis?

ileostomy

will be liquidy

stage 2-4

will require chemotherapy

modifiable risk factors of colorectal cancer

- obesity/physical inactivity - alcohol consumption - smoking - high consumption of red or processed meat - low calcium intake - very low intake of whole-grain fiber, fruit and vegetables

Type I Diabetes Mellitus

"juvenile onset" or "insulin-dependent" diabetes, accounts for approximately 5% of all people with diabetes. Generally affects people younger than 40 years of age, and 40% develop it before 20 years of age.

non-small cell carcinoma- large cell undifferentiated

- 10-15% of all lung cancers - very fast progression - high incidence of metastases - poor prognosis

manifestations of superior vena cava syndrome

- Edema of neck and face Chemotherapy - Headache Radiation - Dizziness and syncope - Visual disturbances - Periorbital edema

superior vena cava syndrome

- Either a partial or complete obstruction of the superior vena cava. - Obstructed venous flow from the head and neck which produce the symptoms

paraneoplastic syndromes

- Seen predominantly in small cell carcinoma - Tumors produce bioactive substances that produce manifestations at sites that are not directly affected by the tumor - ex: ACTH, hypercalcemia, antidiuretic hormone

types of non-small cell carcinoma

- adenocarcinoma - squamous cell - large-cell

non-modifiable risk factors of colorectal cancer

- age above 50 - family/personal history of colon cancer - genetic alterations (familial adenomatous polyposis (FAP)) - Hereditary non-polyposis colorectal cancer (HNPCC) - history of inflammatory bowel disease, ovarian and breast cancers - diabetes - women with a history of endometrial, ovarian, and breast cancer have increased risk

genetic factors of lung cancer

- alterations of tumor suppressor genes - mutations in the EGFR, PD-L1

manifestations of colorectal cancer

- anemia - fatigue - weakness - shortness of breath - exercise intolerance - weight loss

factors decreasing the risk of colorectal cancer

- aspirin - dietary fiber - vitamin D - calcium - oral contraceptives

manifestations of lung cancer

- asymptomatic at first - cough - dyspnea - hemoptysis - chest pain - lymphadenopathy - hoarseness - pneumonia - pancoast syndrome - pleural effusion - pericardial effusion - superior vena cava syndrome - headache - CNS disturbances - GI disturbances - bone pain - hepatomegaly - fever - weakness - fatigue - anorexia - cachexia weight loss - anemia

screening of a high risk patient

- begin age 40 - large or multiple polyps necessitates more frequent follow up - FAP-screen beginning at puberty and colonoscopy yearly - HNPCC-screen at age 20, colonoscopy every 1-2 years - genetic counseling with identified family history

non-small cell carcinoma- squamous cell

- centrally located - earlier symptoms of cough, dyspnea, atelectasis, and wheezing - hypercalemia - metastasize by local invasion - uncommon among nonsmokers

treatment of small cell carcinoma

- chemotherapy/radiation and palliation - rarely surgical interventions

bowel prep

- drink clear liquids the day before - afternoon, they start on bowel prep - cleans out the colon - conscious sedation for colonoscopy

manifestations of pleural effusion

- dyspnea - pain - diminished breath sounds - dull percussion tone

colonoscopy

- every 10 years unless abnormal findings, and then sooner - begin 45 years and end 75 years

flexible sigmoidoscopy

- every 5 years - requires less invasive bowel prep - does not examine the entire colon - can be done without anesthesia

diagnosis of lung cancer

- history and physical exam - chest x-ray - sputum for cytology - CT scan/MRI/PET scan - bronchoscopy/mediastinoscopy - biopsy - bloodwork

complications of surgery

- infection - bowel obstruction/perforation - anastomosis leak - bleeding - bowel perforation - fistula development - ileus

non-small cell lung cancer surgery

- lobectomy - wedge resection - penumonectomy - vats - indicated for stages 1 and 2

lung cancer metastatic sites

- lymph nodes - brain - bones - liver - lung/pleura - adrenal gland

hereditary colon cancers

- make up 5% of colon cancer - familial adenomatous polyposis - Hereditary non-polyposis colorectal cancer (Lynch syndrome)

early manifestations of colorectal cancer

- may be symptomatic - vague abdominal pain - increased flatulence - minor changes is bowel movements with or without bleeding

GI manifestations of colorectal cancer

- melena - hematochezia - anorexia - abdominal pain - feeling of incomplete evacuation of stool - palpable mass - constipation/diarrhea - change in bowel habits - change in the shape of stool - nausea/vomiting - bowel perforation

screening for lung cancer

- no recommendations for the general public - low-dose CT scan for high risk patients

occupational and environmental exposures of lung cancer

- radon - asbestos - cadmium - nickle - second-hand smoke - radiation - air pollution

Preventive measures for hypoglycemia

-Take prescribed dose of medications at proper time -Accurately administer insulin, noninsulin injectables, OA -Ingest all recommended foods at proper times -Provide adequate food intake needed for calories for exercise -Be able to recognize and know symptoms and treat them immediately -Carry simple carbohydrates -Teach family and caregiver about symptoms and treatment - Check blood glucose as ordered -Wear medic alert (diabetic) identification

Type I Diabetes Mellitus - Pathophysiology

-Type I diabetes is an immune-mediated disease caused by autoimmune destruction of the pancreatic B cells, the site of insulin production. This eventually results in a total absence of insulin production. -Autoantibodies to the islet cells cause a reduction of 80% to 90% of normal function before hyperglycemia and other manifestations occur. -A genetic predisposition and exposure to a virus are factors that may contribute to the pathogenesis of immune-related type I diabetes. -In type I diabetes, the islet cell autoantibodies responsible for B cell destruction are present for months to years before the onset of symptoms. Manifestations develop when the person's pancreas can no longer produce sufficient amounts of glucose to maintain normal glucose. Once this occurs, the onset of symptoms is usually rapid. -The patient usually has a history of recent and sudden weight loss and the classic symptoms of polydipsia, polyuria, and polyphagia. -The individual with type I diabetes requires a supply of insulin from an outside source (exogenous insulin) to sustain life. Without insulin, the patient develops diabetic ketoacidosis (DKA), a life-threatening condition resulting in metabolic acidosis.

Onset of Long acting insulin

0.8-4 hours

Onset of intermediate acting insulin

1.5-4 hours

Onset of Rapid acting insulins

10-30 minutes

Onset of inhaled insulin

12-15 minutes

Duration of intermediate acting insulin

12-18 hours

Duration of long acting insulin

16-24 hours

Peak of Short acting insulins

2-5 hours

Duration of inhaled insulin

2.5-3 hours

Peak of intermediate acting insulin

4-12 hours

Duration of short acting insulins

5-8 hours

The doctor ordered NS IVPB of 150 mL to infuse in 40 minutes. Drip Factor is 15 gtt/mL. How many GTT/MIN?

56.25gtt/min (150/40) x 15

The doctor ordered Lanoxin 0.125 mg PO qd. On hand is Lanoxin liquid labeled 0.25 mg per 10 mL. How many mL should be given? Question 52 options:

5mL (.125/.25)x10

The doctor ordered Zidovudine (AZT) 1200 mg PO qd. On hand is Zidovudine 200 mg/ tablet. How many tablet(s) should be given?

6

The nurse explains to a pregnant patient with AIDS that her baby will be treated with antiretroviral drugs for ____________________ weeks after birth.

6

Peak of inhaled insulin

60 minutes

A 60-year-old client comes to the ED reporting weakness, vision problems, increased thirst, increased urination, and frequent infections that do not seem to heal easily. The physician suspects that the client has diabetes. Which classic symptom should the nurse watch for to confirm the diagnosis of diabetes? A. Increased hunger B. Fatigue C. Dizziness D. Numbness

A

A client newly diagnosed with diabetes mellitus asks why he needs ketone testing when the disease affects his blood glucose levels. How should the nurse respond? A "Ketones will tell us if your body is using other tissues for energy." B. "The spleen releases ketones when your body can't use glucose." C. "Ketones can damage your kidneys and eyes." D "Ketones help the physician determine how serious your diabetes is."

A

A client with type 2 diabetes asks the nurse why he can't have a pancreatic transplant. Which of the following would the nurse include as a possible reason? A.Underlying problem of insulin resistance B. Increased risk for urologic complications C. Need for lifelong immunosuppressive therapy D. Need for exocrine enzymatic drainage

A

Which clinical characteristic is associated with type 1 diabetes (previously referred to as insulin-dependent diabetes mellitus)? A. Presence of islet cell antibodies B. Rare ketosis C. Requirement for oral hypoglycemic agents D. Obesity

A

Which is the best nursing explanation for the symptom of polyuria in a client with diabetes mellitus? A. High sugar pulls fluid into the bloodstream, which results in more urine production. B. With diabetes, drinking more results in more urine production. C. Increased ketones in the urine promote the manufacturing of more urine. D. The body's requirement for fuel drives the production of urine.

A

Which statement is correct regarding glargine insulin? A It cannot be mixed with any other type of insulin. B. Its peak action occurs in 2 to 3 hours. C. It is absorbed rapidly. D. It is given twice daily.

A Glargine=lantus=long acting insulin

Which is the primary reason for encouraging injection site rotation in an insulin dependent diabetic? A Promote absorption. B Avoid infection. C. Minimize discomfort. D Prevent muscle destruction.

A Subcutaneous injection sites require rotation to avoid breakdown and/or buildup of subcutaneous fat, either of which can interfere with insulin absorption in the tissue. Infection and discomfort are risks involved with injection site but not the primary reason for rotation of sites. Insulin is not injected into the muscle.

A 34-year-old client comes to the clinic with concerns about an enlarged left testicle and heaviness in his lower abdomen. Which diagnostic test would the nurse expect to be ordered to confirm testicular cancer? a. Alpha-fetoprotein (AFP) b. Prostate-specific antigen (PSA) c. Prostate acid phosphatase (PAP) d. C-reactive protein (CRP)

A AFP is a glycoprotein that is elevated in testicular cancer. PSA and PAP testing is used in the screening of prostate cancer. CRP is diagnostic for inflammatory conditions.

A nurse working with clients who experience alopecia knows that which is the best method of helping clients manage the psychosocial impact of this problem? a. Assisting the client to pre-plan for this event b. Reassuring the client that alopecia is temporary c. Teaching the client ways to protect the scalp d. Telling the client that there are worse side effects

A Alopecia does not occur for all clients who have cancer, but when it does, it can be devastating. The best action by the nurse is to teach the client about the possibility and to give the client multiple choices for preparing for this event. Not all clients will have the same reaction, but some possible actions the client can take are buying a wig ahead of time, buying attractive hats and scarves, and having a hairdresser modify a wig to look like the clients own hair. Teaching about scalp protection is important but does not address the psychosocial impact. Reassuring the client that hair loss is temporary and telling him or her that there are worse side effects are both patronizing and do not give the client tools to manage this condition.

A client is receiving external radiation therapy. Which of the following statements indicates to the nurse that the client understands the discharge teaching plan? a. I need to protect the area from sunlight b. I'm going to apply skin cream every day to the area c. I'll massage the area once a day d. I will wash the markings off after each therapy treatment

A Areas of skin that have received radiation should be protected from sunlight. This prevents skin irritation and subsequent breakdown

The nurse working with oncology clients understands that which age-related change increases the older clients susceptibility to infection during chemotherapy? a. Decreased immune function b. Diminished nutritional stores c. Existing cognitive deficits d. Poor physical reserves

A As people age, there is an age-related decrease in immune function, causing the older adult to be more susceptible to infection than other clients. Not all older adults have diminished nutritional stores, cognitive dysfunction, or poor physical reserves.

A nurse in the oncology clinic is providing preoperative education to a client just diagnosed with cancer. The client has been scheduled for surgery in 3 days. What action by the nurse is best? a. Call the client at home the next day to review teaching. b. Give the client information about a cancer support group. c. Provide all the preoperative instructions in writing. d. Reassure the client that surgery will be over soon.

A Clients are often overwhelmed at a sudden diagnosis of cancer and may be more overwhelmed at the idea of a major operation so soon. This stress significantly impacts the clients ability to understand, retain, and recall information. The nurse should call the client at home the next day to review the teaching and to answer questions. The client may or may not be ready to investigate a support group, but this does not help with teaching. Giving information in writing is important (if the client can read it), but in itself will not be enough. Telling the client that surgery will be over soon is giving false reassurance and does nothing for teaching.

A client is placed on a medical regimen of doxorubicin (Adriamycin), cyclophosphamide (Cytoxan), and fluorouracil (5-FU) for breast cancer. Which side effect seen in the client should the nurse report to the provider immediately? a. Shortness of breath b. Nausea and vomiting c. Hair loss d. Mucositis

A Doxorubicin (Adriamycin) can cause cardiac problems with symptoms of extreme fatigue, shortness of breath, chronic cough, and edema. These need to be reported as soon as possible to the provider. Nausea, vomiting, hair loss, and mucositis are common problems associated with chemotherapy regimens.

A nurse works on an oncology unit and delegates personal hygiene to an unlicensed assistive personnel (UAP). What action by the UAP requires intervention from the nurse? a. Allowing a very tired client to skip oral hygiene and sleep b. Assisting clients with washing the perianal area every 12 hours c. Helping the client use a soft-bristled toothbrush for oral care d. Reminding the client to rinse the mouth with water or saline

A Even though clients may be tired, they still need to participate in hygiene to help prevent infection. The other options are all appropriate.

The nurse is teaching a 45-year-old woman about her fibrocystic breast condition. Which statement by the client indicates a lack of understanding? a. This condition will become malignant over time. b. I should refrain from using hormone replacement therapy. c. One cup of coffee in the morning should be enough for me. d. This condition makes it more difficult to examine my breasts.

A Fibrocystic breast condition does not increase a womans chance of developing breast cancer. Hormone replacement therapy is not indicated since the additional estrogen may aggravate the condition. Limiting caffeine intake may give relief for tender breasts. The fibrocystic changes to the breasts make it more difficult to examine the breasts because of fibrotic changes and lumps.

A client is receiving interleukins along with chemotherapy. What assessment by the nurse takes priority? a. Blood pressure b. Lung assessment c. Oral mucous membranes d. Skin integrity

A Interleukins can cause capillary leak syndrome and fluid shifting, leading to intravascular volume depletion. Although all assessments are important in caring for clients with cancer, blood pressure and other assessments of fluid status take priority.

A client is receiving chemotherapy through a peripheral IV line. What action by the nurse is most important? a. Assessing the IV site every hour b. Educating the client on side effects c. Monitoring the client for nausea d. Providing warm packs for comfort

A Intravenous chemotherapy can cause local tissue destruction if it extravasates into the surrounding tissues. Peripheral IV lines are more prone to this than centrally placed lines. The most important intervention is prevention, so the nurse should check hourly to ensure the IV site is patent, or frequently depending on facility policy. Education and monitoring for side effects such as nausea are important for all clients receiving chemotherapy. Warm packs may be helpful for comfort, but if the client reports that an IV site is painful, the nurse needs to assess further.

A client is diagnosed with metastatic prostate cancer. The client asks the nurse the purpose of his treatment with the luteinizing hormonereleasing hormone (LH-RH) agonist leuprolide (Lupron) and the bisphosphonate pamidronate (Aredia). Which statement by the nurse is most appropriate? a. The treatment reduces testosterone and prevents bone fractures. b. The medications prevent erectile dysfunction and increase libido. c. There is less gynecomastia and osteoporosis with this drug regimen. d. These medications both inhibit tumor progression by blocking androgens.

A Lupron, an LH-RH agonist, stimulates the pituitary gland to release luteinizing hormone (LH) to the point that the gland is depleted of LH and testosterone production is lessened. This may decrease the prostate cancer since it is hormone dependent. Lupron can cause osteoporosis, which results in the need for Aredia to prevent bone loss. Erectile dysfunction, decreased libido, and gynecomastia are side effects of the LH-RH medications. Antiandrogen drugs inhibit tumor progression by blocking androgens at the site of the prostate.

A woman diagnosed with breast cancer had these laboratory tests performed at an office visit: Alkaline phosphatase 125 U/L Total calcium 12 mg/dL Hematocrit 39% Hemoglobin 14 g/dL Which test results indicate to the nurse that some further diagnostics are needed? a. Elevated alkaline phosphatase and calcium suggests bone involvement. b. Only alkaline phosphatase is decreased, suggesting liver metastasis. c. Hematocrit and hemoglobin are decreased, indicating anemia. d. The elevated hematocrit and hemoglobin indicate dehydration.

A The alkaline phosphatase (normal value 30 to 120 U/L) and total calcium (normal value 9 to 10.5 mg/dL) levels are both elevated, suggesting bone metastasis. Both the hematocrit and hemoglobin are within normal limits for females.

A client hospitalized for chemotherapy has a hemoglobin of 6.1 mg/dL. What medication should the nurse prepare to administer? a. Epoetin alfa (Epogen) b. Filgrastim (Neupogen) c. Mesna (Mesnex) d. Oprelvekin (Neumega)

A The clients hemoglobin is low, so the nurse should prepare to administer epoetin alfa, a colony-stimulating factor that increases production of red blood cells. Filgrastim is for neutropenia. Mesna is used to decrease bladder toxicity from some chemotherapeutic agents. Oprelvekin is used to increase platelet count.

A client is discharged to home after a modified radical mastectomy with two drainage tubes. Which statement by the client would indicate that further teaching is needed? a. I am glad that these tubes will fall out at home when I finally shower. b. I should measure the drainage each day to make sure it is less than an ounce. c. I should be careful how I lie in bed so that I will not kink the tubing. d. If there is a foul odor from the drainage, I should contact my doctor.

A The drainage tubes (such as a Jackson-Pratt drain) lie just under the skin but need to be removed by the health care professional in about 1 to 3 weeks at an office visit. Drainage should be less than 25 mL in a days time. The client should be aware of her positioning to prevent kinking of the tubing. A foul odor from the drainage may indicate an infection; the doctor should be contacted immediately.

A client with a history of prostate cancer is in the clinic and reports new onset of severe low back pain. What action by the nurse is most important? a. Assess the client's gait and balance. b. Ask the client about the ease of urine flow. c. Document the report completely. d. Inquire about the client's job risks.

A This client has manifestations of spinal cord compression, which can be seen with prostate cancer. This may affect both gait and balance and urinary function. For client safety, assessing gait and balance is the priority. Documentation should be complete. The client may or may not have occupational risks for low back pain, but with his history of prostate cancer, this should not be where the nurse starts investigating.

A sibling of a client recently diagnosed with colon cancer questions the nurse in regard to the carcinoembryonic antigen (CEA) blood test. Appropriate information from the nurse should include, "the CEA test is a. most helpful in monitoring the progress of the disease in clients already being treated for colon cancer b. an effective screening test and is indicated due to your family history of colon cancer c. recommended by the American Cancer Association to be performed yearly starting at age 50 d. Used to confirm the diagnosis if a client has symptoms consistent with colon or rectal cancer.

A CEA is a substance produced by cells of most colon and rectal cancers and released into the blood stream. CEA levels should return to normal following successful treatment. If follow-up tests detect a steady increase in CEA levels, additional testing is recommended to find out if the cancer, it is not used as a screening test for colon cancer in clients who have not been diagnosed with cancer and appear to be healthy

A client with leukemia suffers a relapse, and the provider recommends a bone marrow transplant. After the provider leaves, the client asks the nurse, "do they put the marrow back in me the same way they took it out for a biopsy?" Which of the following responses should the nurse make? a. No, it's transfused just like any other blood component into your central IV line b. Yes, it's replaced directly into the bone like a biopsy, but it requires several sites c. Yes, however, you will not feel it because they will do it in the OR under anesthesia d. I'll get the doctor back in here to answer the rest of your questions

A Harvested marrow is transfused into the recipient like any other blood component via the client's central IV line

A client who has a suspected ovarian tumor is scheduled to have a pelvic ultrasound. In preparing the client for the procedure, the nurse instructs the client to a. drink 4-6 glasses of water and do not void b. have nothing to eat or drink after midnight c. self-administer an enema the night before the procedure d. abstain from sexual intercourse the night before

A The ultrasound machine sends out high-frequency sound waves which bounce off body structures to create a picture. A pelvic ultrasound requires a full bladder to batter visualize and identify the organs and structures in the pelvis. To accomplish this, the nurse should instruct the client to drink 4-6 full glasses of water just prior to the procedure and not to void.

Diabetes

A chronic multisystem disease r/t abnormal insulin production, impaired insulin utilization, or both Leading cause of adult blindness, end-stage kidney disease, and nontraumatic lower limb amputations. It is also a major contributing factor to heart disease and stroke. The two most common types are Type I and Type II classifications.

A nurse is providing dietary instructions to a client with hypoglycemia. To control hypoglycemic episodes, the nurse should recommend: A. eating a candy bar if light-headedness occurs. B.increasing intake of vitamins B and D and taking iron supplements. C. consuming a low-carbohydrate, high-protein diet and avoiding fasting. D. increasing saturated fat intake and fasting in the afternoon.

C

IGT (impaired glucose tolerance)

A diagnosis is made if the 2 hour oral glucose tolerance test (OGTT) values are 140-199 mg/dL or 7.8 to 11.0 mmol/L.

A woman has been using acupuncture to treat the nausea and vomiting caused by the side effects of chemotherapy for breast cancer. Which conditions would cause the nurse to recommend against further use of acupuncture? (Select all that apply.) a. Lymphedema b. Bleeding tendencies c. Low white blood cell count d. Elevated serum calcium e. High platelet count

A, B, C Acupuncture could be unsafe for the client if there is poor drainage of the extremity with lymphedema or if there was a bleeding tendency and low white blood cell count. Coagulation would be compromised with a bleeding disorder, and the risk of infection would be high with the use of needles. An elevated serum calcium and high platelet count would not have any contraindication for acupuncture.

A nurse is preparing to administer IV chemotherapy. What supplies does this nurse need? (Select all that apply.) a. Chemo gloves b. Facemask c. Isolation gown d. N95 respirator e. Shoe covers

A, B, C The Occupational Safety and Health Administration (OSHA) and the Oncology Nurses Society have developed safety guidelines for those preparing or administering IV chemotherapy. These include double gloves (or chemo gloves), a facemask, and a gown. An N95 respirator and shoe covers are not required.

A clients family members are concerned that telling the client about a new finding of cancer will cause extreme emotional distress. They approach the nurse and ask if this can be kept from the client. What actions by the nurse are most appropriate? (Select all that apply.) a. Ask the family to describe their concerns more fully. b. Consult with a social worker, chaplain, or ethics committee. c. Explain the clients right to know and ask for their assistance. d. Have the unit manager take over the care of this client and family. e. Tell the family that this secret will not be kept from the client.

A, B, C The clients right of autonomy means that the client must be fully informed as to his or her diagnosis and treatment options. The nurse cannot ethically keep this information from the client. The nurse can ask the family to explain their concerns more fully so everyone understands the concerns. A social worker, chaplain, or ethics committee can become involved to assist the nurse, client, and family. The nurse should explain the clients right to know and ask the family how best to proceed. The nurse should not abdicate responsibility for this difficult situation by transferring care to another nurse. Simply telling the family that he or she will not keep this secret sets up an adversarial relationship. Explaining this fact along with the concept of autonomy would be acceptable, but this by itself is not.

A nurse is providing community education on the seven warning signs of cancer. Which signs are included? (Select all that apply.) a. A sore that does not heal b. Changes in menstrual patterns c. Indigestion or trouble swallowing d. Near-daily abdominal pain e. Obvious change in a mole

A, B, C, E The seven warning signs for cancer can be remembered with the acronym CAUTION: changes in bowel or bladder habits, a sore that does not heal, unusual bleeding or discharge, thickening or lump in the breast or elsewhere, indigestion or difficulty swallowing, obvious change in a wart or mole, and nagging cough or hoarseness. Abdominal pain is not a warning sign.

A client has thrombocytopenia. What actions does the nurse delegate to the unlicensed assistive personnel (UAP)? (Select all that apply.) a. Apply the clients shoes before getting the client out of bed. b. Assist the client with ambulation. c. Shave the client with a safety razor only. d. Use a lift sheet to move the client up in bed. e. Use the Waterpik on a low setting for oral care.

A, B, D Clients with thrombocytopenia are at risk of significant bleeding even with minor injuries. The nurse instructs the UAP to put the clients shoes on before getting the client out of bed, assist with ambulation, shave the client with an electric razor, use a lift sheet when needed to reposition the client, and use a soft-bristled toothbrush for oral care.

The nurse is describing the action of insulin in the body to a client newly diagnosed with type 1 diabetes. Which of the following would the nurse explain as being the primary action? A. It aids in the process of gluconeogenesis. B It stimulates the pancreatic beta cells. C It enhances the transport of glucose across the cell membrane. D It decreases the intestinal absorption of glucose.

C

A client has mucositis. What actions by the nurse will improve the clients nutrition? (Select all that apply.) a. Assist with rinsing the mouth with saline frequently. b. Encourage the client to eat room-temperature foods. c. Give the client hot liquids to hold in the mouth. d. Provide local anesthetic medications to swish and spit. e. Remind the client to brush teeth gently after each meal.

A, B, D, E Mucositis can interfere with nutrition. The nurse can help with rinsing the mouth frequently with water or saline; encouraging the client to eat cool, slightly warm, or room-temperature foods; providing swish-and-spit anesthetics; and reminding the client to keep the mouth clean by brushing gently after each meal. Hot liquids would be painful for the client.

The nurse is taking a history of a 68-year-old woman. What assessment findings would indicate a high risk for the development of breast cancer? (Select all that apply.) a. Age greater than 65 years b. Increased breast density c. Osteoporosis d. Multiparity e. Genetic factors

A, B, E The high risk factors for breast cancer are age greater than 65 with the risk increasing until age 80; an increase in breast density because of more glandular and connective tissue; and inherited mutations of BRCA1 and/or BRCA2 genes. Osteoporosis and multiparity are not risk factors for breast cancer. A high postmenopausal bone density and nulliparity are moderate and low increased risk factors, respectively.

After a breast examination, the nurse is documenting assessment findings that indicate possible breast cancer. Which abnormal findings need to be included as part of the clients electronic medical record? (Select all that apply.) a. Peau d'orange b. Dense breast tissue c. Nipple retraction d. Mobile mass at two o'clock e. Nontender axillary nodes

A, C, D In the documentation of a breast mass, skin changes such as dimpling (peau d'orange), nipple retraction, and whether the mass is fixed or movable are charted. The location of the mass should be stated by the face of a clock. Dense breast tissue and nontender axillary nodes are not abnormal assessment findings that may indicate breast cancer.

A client receiving chemotherapy has a white blood cell count of 1000/mm3. What actions by the nurse are most appropriate? (Select all that apply.) a. Assess all mucous membranes every 4 to 8 hours. b. Do not allow the client to eat meat or poultry. c. Listen to lung sounds and monitor for cough. d. Monitor the venous access device appearance with vital signs. e. Take and record vital signs every 4 to 8 hours.

A, C, D, E Depending on facility protocol, the nurse should assess this client for infection every 4 to 8 hours by assessing all mucous membranes, listening to lung sounds, monitoring for cough, monitoring the appearance of the venous access device, and recording vital signs. Eating meat and poultry is allowed.

The nurse is administering an insulin drip to a patient in ketoacidosis. What insulin does the nurse know can be used intravenously? Select all that apply. A. Rapid-acting B. Intermediate-acting C. Long-acting D. Short-acting

A, D Insulins may be grouped into several categories based on the onset, peak, and duration of action. Rapid- and short-acting insulin can be administered by IV

The nurse is formulating a teaching plan according to evidence-based breast cancer screening guidelines for a 50-year-old woman with low risk factors. Which diagnostic methods should be included in the plan? (Select all that apply.) a. Annual mammogram b. Magnetic resonance imaging (MRI) c. Breast ultrasound d. Breast self-awareness e. Clinical breast examination

A, D, E Guidelines recommend a screening annual mammogram for women ages 40 years and older, breast self- awareness, and a clinical breast examination. An MRI is recommended if there are known high risk factors. A breast ultrasound is used if there are problems discovered with the initial screening or dense breast tissue.

A client is interested in learning about the risk factors for prostate cancer. Which factors does the nurse include in the teaching? (Select all that apply.) a. Family history of prostate cancer b. Smoking c. Obesity d. Advanced age e. Eating too much red meat f. Race

A, D, E, F Advanced family history of prostate cancer, age, a diet high in animal fat, and race are all risk factors for prostate cancer. Smoking and obesity are not known risk factors.

A young gay patient being treated for his third sexually transmitted disease does not see why he should use condoms, because "they don't work." The nurse's most appropriate response would be A. "Condoms may not provide 100% protection, but when used correctly and consistently with every act of sexual intercourse they reduce your risk of getting infected with HIV or other sexually transmitted diseases." B. "Condoms do not provide 100% protection, but when used with a spermicide you can be assured of complete protection against HIV and other STDs." C. "You are correct. Condoms don't always work, so your best protection is to limit your number of partners." D. "Condoms do not provide 100% protection, so you should always discuss with your sexual partners their HIV status or ask if they have any STD."

A. "Condoms may not provide 100% protection, but when used correctly and consistently with every act of sexual intercourse they reduce your risk of getting infected with HIV or other sexually transmitted diseases."

A patient asks the nurse, "How does HIV cause AIDS?" The nurse's response should be A. "HIV attacks the immune system, a system that protects the body from foreign invaders, making it unable to protect the body from organisms that cause diseases." B. "HIV attacks the digestive system, decreasing the absorption of essential nutrients and causing weight loss and fatigue." C. "HIV attacks the respiratory system, making the lungs more susceptible to organisms that cause pneumonia." D. "HIV breaks down the cir

A. "HIV attacks the immune system, a system that protects the body from foreign invaders, making it unable to protect the body from organisms that cause diseases."

A client is suspected of having a disorder of the adrenal medulla. The provider has ordered a vanillylmandelic acid (VMA) test. Which of the following are appropriated reinforcements by the nurse? A. "You will need to collect your urine for 24 hours." B. "Be sure to drink 8 ounces of water before your first void." C. "Minimize your activity during the test." D. "Avoid bananas and chocolate 2 days before the test." E. "Don't take your antihypertensive medication during the test."

A. CORRECT: VMA testing includes 24 hr urine collection. B. INCORRECT: Clients can consume fluids at their own discretion. C. INCORRECT: VMA testing does not restrict the client's activity. D. CORRECT: Bananas, caffeine, vanilla, and chocolate should be avoided 2 to 3 days prior to testing. E. CORRECT: Antihypertensives should be discontinued during testing.

Members of the local emergency medical service unit have just transported an accident victim to the emergency department. They tell the ED nurse that the victim has AIDS, and they have blood on their hands and clothing. The nurse advises them to wash their hands and to change any clothing that is wet with blood. The nurse talks to them about getting a baseline HIV test and about future testing. They ask the nurse how long it will take before they will know if they are infected. The nurse's response should be A. "You will need to be tested in 3 months and again in 6 months, because 95% of people seroconvert to antibody-positive in 3 months and 99% in 6 months." B. "You will need to be tested in 6 weeks and regularly for the next 10 years, because that is the average length of time it takes to detect the virus by testing." C. "Transmission by occupational exposure is rare. There is no need for you to be concerned." D. "You will need to be tested in 3 weeks, because most people seroconvert in that length of time."

A. "You will need to be tested in 3 months and again in 6 months, because 95% of people seroconvert to antibody-positive in 3 months and 99% in 6 months."

The physician has ordered an outpatient dexamethasone suppression test to diagnose the cause of Cushing syndrome in a patient who works at night from 11:00 pm to 7:00 am and normally sleeps from 8:00 am to 4:00 pm. The patient has been given the dexamethasone; to ensure the most reliable test results, the nurse arranges for the plasma cortisol level to be drawn at which of the following times? a) 5:00 pm b) 8:00 pm c) 8:00 am d) 12:00 pm

A. An overnight dexamethasone suppression test is used to diagnosis pituitary and adrenal causes of Cushing syndrome. It can be performed on an outpatient basis. Dexamethasone is administered orally late in the evening or at bedtime, and a plasma cortisol level is obtained at 8 am the next morning. However, in a patient who sleeps during the day, the medication would be given before bed and the plasma level would be drawn soon after awakening in the late afternoon.

The nurse clarifies that a patient with HIV does not necessarily have AIDS until: A. CD4 cell level drops to 200. B. Two or more opportunistic infections are diagnosed. C. Kaposi sarcoma appears. D. Tested positive for enzyme-linked immunosorbent assay (ELISA).

A. CD4 cell level drops to 200.

A nurse is reviewing the laboratory findings of a client who has cushing's disease. Which of the following findings are expected for this client? (Select all that apply). A. Sodium 150 mEq/L B. Potassium 3.3 mEq/L C. Calcium 8.0 mg/dL D. Lymphocyte count 35% E. Fasting glucose 145 mg/dL

A. CORRECT: Hypernatremia is an expected finding for clients who have Cushing's disease. B. CORRECT: Hypokalemia is an expected finding for clients who have Cushing's disease. C. CORRECT: Hypocalcemia is an expected finding for clients who have Cushing's disease D. INCORRECT: A decreased lymphocyte count is an expected finding for clients who have Cushing's disease. E. CORRECT: Clients who have Cushing's disease have an elevated fasting glucose because glucose metabolism is affected.

A nurse is reviewing serum laboratory results for a client who has Addison's disease. Which of the following findings are typical for a client who has this condition? (select all that apply.) A. Sodium 130 mEq/L B. Potassium 6.1 mEq/L C. Calcium 11.6 mg/dL D. Magnesium 2.5/dL E. Glucose 65 mg/dL

A. CORRECT: In the presence of Addison's disease, insufficient glucose can cause sodium and water excretion. Therefore, this is an expected finding. B CORRECT: Hyperkalemia is an expected finding for a client who has Addison's disease. C. CORRECT: Hypercalcemia is an expected finding for a client who has Addison's disease. D. INCORRECT: Although this finding is above the expected reference range, it is not an expected finding for a client who has Addison's disease. E. CORRECT: This finding is below the expected reference range. Hypoglycemia is an expected finding for a client who has Addison's disease.

A nurse in a provider's office is assisting with the plan of care for a client of care for a client who has a new diagnosis of Grave' disease and a new prescription for methimazole (Tapazole). Which of the following should the nurse include in the plan of care? (Select all that apply). A. Monitor CBC B. Monitor triiodothyronine (T3) C. Inform the client that the medication at the same time every day. D. Advise the client to take the medication at the same time every day. E. Inform the client that an adverse effect of this medication is iodine toxicity.

A. CORRECT: Methimazole can cause a number of hematologic effects, including leukopenia and thrombocytopenia. The nurse should monitor the client's CBC. B. CORRECT: MEthimazole reduces thyroid hormone production. The nurse should monitor the client's T3 C. INCORRECT: Methimazole can be prescribed for the client who has Graves' disease for 1 to 2 years. D. CORRECT: Methimazole should be taken at the same time every day to maintain blood levels. E. INCORRECT: Iodine toxicity is an adverse effect of Lugol's solution.

A nurse is reviewing the laboratory findings of a client who has suspected hyperthyroidism. An elevation of which of the following supports this diagnosis? A. Triiodothyronine (T3) B. Vanillylmandelic (VMA) C. Adrenocorticoticotropic hormone (ACTH) D. Catecholamines

A. CORRECT: T3 increases in a hyperthyroid state. B. INCORRECT: VMA is used to detect pheochromocytoma and reflects the amount of catecholamine byproducts. C.INCORRECT: ACTH is used to detect Cushing's disease. D. INCORRECT: Catecholamines are measured following clonidine (Catapres) administration to diagnose pheocromocytoma.

A nurse is preparing to receive a client from the PACU who is postoperative following a thyroidectomy. The nurse should ensure that which of the following equipment is available? (Select all that apply.) A. Suction equipment B. Humidified air C. Flashlight D. Tracheostomy tray E. Oxygen delivery equipment

A. CORRECT: The client can require oral or tracheal suctioning. The nurse should ensure that this equipment is available. B. CORRECT: Humidified air thins secretions and promotes respiratory exchange. This equipment should be available. C. INCORRECT: A flashlight is used to measure the reaction of the pupils to light for a client who has an intracranial disorder. D. CORRECT: The client can experience respiratory obstruction. E. CORRECT: The client can require supplemental oxygen due to respiratory complication. This equipment should be availiable

Before the initiation of any anti-HIV drug protocol, the patient is assessed for the willingness to: A. Comply with drug protocol. B. Involve the partner in a support program. C. Give up sexual activity for several months. D. Follow the strict dietary guidelines.

A. Comply with drug protocol.

The nurse caring for a patient with HIV infection who is taking Retrovir, a nucleoside antiviral that is a reverse transcriptase inhibitor, should be especially observant for side effects of: A. Granulocytopenia B. Hypertensive episodes C. Decreased urine output D. Edema of the face

A. Granulocytopenia

A nurse in a provider's office is reviewing the laboratory findings of a client who is being evaluated for primary hypothyroidism. Which of the following laboratory findings is indicative of this condition? A. Serum t4 10 mcg/dL B. Serum t3 200 ng/dL C. Hematocrit 34% D. Serum cholesterol 180 mg/dL

A. INCORRECT: A serum t4 of 10 mcg/dL is within the expected reference range. A decreased serum t4 is an expected finding for a client who has hypothyroidism B. INCORRECT: Serum T3 of 200 ng/mL is within the expected reference range. A decreased serum t3 is an expected finding for a client who has hyopthyroidism C. CORRECT: Hematocrit of 34% indicates anemia, which is an expected result for a client who has hyopthyroidism. D. INCORRECT: Serum cholesterol of 180 mg/dL is within the expected reference range. An elevated serum cholesterol is an expected finding for a client who has hypothyroidism.

A client asks a nurse why the provider bases his medication regimen on his HbA1c instead of his log of morning fasting blood glucose results. Which of the following is an appropriate response by the nurse? A."HbA1c measures how well insulin is regulation you blood glucose between meals." B."HbA1c indicates how well your blood glucose has been regulated over the past three months." C."A test of HbA1c is the first test to determine if an individual has diabetes D."A test of HbA1c determines if the dosage of insulin needs to be adjusted."

A. INCORRECT: Capillary glucose monitoring evaluates how well insulin is regulating blood glucose between meals. B. CORRECT: HbA1c measures blood glucose control over the past 2 to 3 months. C. INCORRECT: A fasting blood glucose is the first test performed to diagnose diabetes mellitus. D. INCORRECT: Capillary glucose monitoring evaluates how well insulin regulates blood glucose.

A nurse is in a long-term care facility is assisting with the plan of care for a client who has hypothyroidism. Which of the following should the nurse recommend for the client's plan of care? (Select all that apply.) A. Apply heating pad to lower legs to provide warmth. B. Provide rest periods throughout the day. C. Provide a high calorie diet. D. Elevate the client's legs when in chair. E. Apply emollient lotion to the client's skin after bathing.

A. INCORRECT: Clinical manifestations of hypothyroidism include vasodilation, decreased sensation, and decreased alertness. The nurse should avoid the use of electric heating devices because it places the client at risk for burns. B. CORRECT: A clinical manifestation of hypothyroidism is fatigue. The nurse should plan to provide rest periods throughout the day. C. INCORRECT: Weight gain is a clinical manifestation of hypothyroidism. The nurse should provide the client with a low-calorie diet. D. CORRECT: A clinical manifestation of hypothyroidism is edema. The nurse should elevate the client's legs when he is sitting in a chair to promote venous return E. CORRECT: Dry skin is a clinical manifestation of hypothyroidism. The nurse should apply an alcohol-free lotion emollient lotion to the client's skin after bathing.

A nurse is reinforcing teaching with a client who is scheduled for a phentolamine blocking test. This test supports a diagnosis of which of the following disorders? A. Addison's disease B. diabetes mellitus C. Cushing's disease D. Pheochromocytoma

A. INCORRECT: Evaluation of plasma cortisol is used to identify addison's disease. B. INCORRECT: A fasting blood glucose is used to identify diabetes mellitus. C. INCORRECT: an dexamethoasone suprresion test is used to identify Cushing's syndrome. CORRECT: Phentolamine, an alpha blocker, is administered and decreases blood pressure when pheochromocytoma is present.

A nurse in a provider's office is reviewing the health record of a client who is being evaluated for Graves' disease. Which of the following is an expected laboratory finding for this client? A. Decreased thyrotropin antibodies B. Decreased thyroid stimulating hormone C. Decreased free thyroxine index D. Decreased triiodothyronine

A. INCORRECT: In the presence of Graves' disease, elevated thyrotropin receptor antibodies is an expected finding. B. CORRECT: In the presence of Graves' disease, low thyroid stimulating hormone (TSH), is an expected finding. The pituitary gland decreases the production of TSH when thyroid hormone levels are elevated. C. INCORRECT: In the presence of Graves' disease, elevated free thyroxine index is an expected finding. D. INCORRECT: In the presence of GRAVE's disease, elevated triodothyronine is an expected finding.

A nurse is reinforcing teaching with a client who has been prescribed levothyroxine (Synthroid) to treat hypothyroidism. Which of the following should the nurse include in the teaching? (Select all that apply). A. Weight gain is expected while taking this medication B. Medication should not be discontinued without the advice of the provider. C. Follow-up serum TSH levels should be obtained. D. Take the medication on an empty stomach. E. Use fiber laxatives for constipation

A. INCORRECT: Levothyroxine speeds up metabolism. Therefore, weight loss is an expected effect. B. CORRECT: The provider carefully titrates the dosage of this medication. It should be increased slowly until the client reaches an euthyroid state. The client should not discontinue the medication unless directed to do so by the provider. C. CORRECT: Serum TSH levels are used to measure the effectiveness of the medication. D. CORRECT: Serum TSH levels are used to measure the effectiveness of the medication E. INCORRECT: Fiber laxatives reduce absorption of the medication and should be avoided.

A nurse is reviewing the clinical manifestations of hyperthyroidism with a client. Which of the following findings should the nurse include? (Select all that apply) A. Dry skin B. Heat intolerance C. Constipation D. Palpitations E Weight loss F. Bradycardia

A. INCORRECT: Moist skin is an expected finding for the client who has hyperthyroidism. B. CORRECT: HYperthyroidism increases the client's metabolism. Heat intolerance is an expected finding. C. INCORRECT: Diarrhea is an expected finding for the client who has hyperthyroidism. D. CORRECTl Hyperthroidism increases the client's metabolism. Weight loss is an expected finding for the client who has hyperthyroidism. E. CORRECT: Hyperthyroidism increases the client's metabolism. Weight loss is an expected finding for the client who has hyperthyroidism. F. INCORRECT: Hyperthyroidism increases the client's metabolism. Tachycardia is an expected finding for the client who has hyperthyroidism.

A nurse is caring for a patient suspected of having a pituitary tumor causing panhypopituitarism. During assessment of the patient, which of the following clinical manifestations would the nurse expect to find? a) Atrophy of the gonads b) Carpopedal spasm c) Hypertension d) Tachycardia

A. Undersecretion (hyposecretion) commonly involves all of the anterior pituitary hormones and is termed panhypopituitarism. In this condition, the thyroid gland, the adrenal cortex, and the gonads atrophy (shrink) because of loss of the tropic-stimulating hormones.

A nurse is planning to teach a client who is being evaluated for Addison's disease about the adrenocorticotropic hormone (ACTH) stimulation test. The nurse should base her instructions to the client on which of the following? A. The ACTH stimulation test measures the response by the kidneys to ACTH. B. In the presence of primary adrenal insufficiency, plasma cortisol levels rise in response to administration of ACTH. C. ACTH is a hormone produced by the pituaitary gland. D. The client is instructed to take a dose of ACTH by mouth the evening before the test.

A. INCORRECT: The ACTH stimulation test measures the response by the adrenal glands to ACTH. B. INCORRECT: In the presence of primary adrenal insufficiency, plasma cortisol levels do not rise in response to administration of ACTH. C. CORRECT: Secretion of corticotropin-releasing hormone from the hypothalamus prompts the pituitary gland to secrete ACTH. D. INCORRECT: ACTH is administered IV during the testing process, and plasma cortisol levels are measured 30 min and 1 hr after the injection

A nurse is reinforcing discharge instructions to a client who had a transsphenoidal hypophysectoemy. Which of the following instructions should the nurse include? (Select all that apply). A. Brush teeth after every meal or snack. B. Avoid bending at the knees. C. Eat a high-fiber diet. D. Notify the provider of any sweet-tasting drainage. E. Notify the provider if he has diminished sense of smell.

A. INCORRECT: The client should avoid brushing his teeth for 2 weeks to allow time for the incision to heal. B. INCORRECT: The client should avoid bending at the waist. If bending is necessary, he should bend at the knees. C. CORRECT: To avoid constipation, which contributes to increased intracranial pressure, the client should eat a high-fiber diet. Docusate sodium (Colace) can be used to prevent constipation. D. CORRECT: Sweet tasting fluid is an indication of a cerebral spinal fluid leak. The client should notify the provider. E. INCORRECT: Diminshed sense of smell is an expected finding after the surgery.

A nurse is caring for a client who is 36 hr postoperative following a tanssphenoidal hypophysectomy. The nurse should test the client's nasal drainage for the presence of which of the following? A. RBCs B. Ketones C. Glucose D. Streptococcus

A. INCORRECT: The nurse should not test for the presence of RBC as an indication of cerebral spinal fluid leak. B. INCORRECT: The nurse should not test for the presence of Ketones as an indication of cerebral spinal fluid leak. C. CORRECT: Cerebral spinal fluid contains glucose. Therefore, the nurse should test nasal drainage for glucose to determine whether the nasal drainage contains glucose D. INCORRECT: The nurse should not test for the presence of streptococcus as an indication of cerebral spinal fluid leak

A nurse is collecting data on a client who is 12 hr postoperative following a thyroidectomy. Which of the following findings are indicated of thyroid crisis? (Select all that apply.) A. Bradycardia B. Hypothermia C. Tremors D. Abdominal pain E. Mental confusion

A. INCORRECT: When thyroid crisis occurs, the client experiences an extreme rise in metabolic rate, which results in tachycardia. B. INCORRECT: When thyroid crisis occurs, the client experiences an extreme rise in metabolic rate, which results in a high fever. C. CORRECT: Excessive levels of thyroid hormone can cause the client to experience tremors. D. CORRECT: When thyroid crisis occurs, the client can experience gastrointestinal conditions, such as vomiting. diarrhea, and abdominal pain. E. CORRECT: Excessive thyroid hormone levels can cause the client to experience mental confusion

A nurse is providing instructions to a client who has Graves' disease and has a new prescription for propranolol (Inderal). Which of the following information should the nurse include? A. An adverse effect of this medication is jaundice. B. Take your pulse before each dose. C. The purpose of this medication is to decrease production of thyroid hormone. D. You should stop taking this medication if you have a sore throat.

A. INCORRECT: Yellowing of the skin is an adverse effect of methimazole. B. CORRECT: Propranolol can cause bradycardia. The client should take his pulse before each dose. If there is a significant change, he should withhold the dose and consult his provider. C. INCORRECT: The purpose of this medication is to suppress tachycardia, diaphoresis, and other effects of Graves' disease. D. INCORRECT: sore throat is not an adverse effect of this medication. THe client should not discontinue taking this medication because this action can result in tachycardia and dysrhythias.

The nursing diagnosis that would take priority in the care of an outpatient with AIDS would be: A. Ineffective therapeutic regimen management B. Impaired physical mobility C. Social isolation D. Impaired skin integrity

A. Ineffective therapeutic regimen management

Long-term use of antithyroid medication is not generally recommended for elderly patients because of which of the following events? a) Agranulocytosis and hepatic injury b) Renal disease and mental confusion c) GI complications and weight loss d) Cardiac arrhythmias and fatigue

A. Long-term use of certain antithyroid medications, such as propylthiouracil (PTU), is not recommended for treatment of toxic nodular goiter in older patients due to the risk of side effects. Although rare, there is evidence that PTU can result in agranulocytosis and hepatic injury. However, use of antithyroid medications versus radioactive iodine or surgery may be the patient's preferred choice or the option for some older patients and other ill persons with "limited longevity" who can be monitored at least every 3 months.

The nurse takes into consideration in planning care for a patient with acquired immunodeficiency syndrome (AIDS) that the common form of transmission of the HIV virus is: A. Male to male B. Exposure to contaminated blood products C. Heterosexual contact D. Injection drug use

A. Male to male

One of the implementations that the nurse could use to increase the comfort of a patient with oral hair leukoplakia is to: A. Offer fluids through a straw. B. Limit intake of ice cream and other cold foods. C. Encourage mouth rinses with warm salt water several times a day. D. Allow aspirin to melt in the mouth and then wash out with warm water.

A. Offer fluids through a straw.

When teaching a patient diagnosed with hypothyroidism regarding medical intervention, which of the following is important to communicate? a) Thyroid hormone (TH) may increase the effect of digitalis preparation. b) Increased resorption occurs with thyroid hormone (TH). c) Thyroid hormone (TH) may decrease blood glucose levels. d) The normal dosages of sedative agents are prescribed.

A. Thyroid hormones may increase the pharmacologic effect of digitalis glycosides, anticoagulant agents, and indomethacin, necessitating careful observation and assessment by the nurse for side effects.

Following a thyroidectomy, a patient develops a carpopedal spasm while the nurse is taking a BP reading on the left arm. Which of the following actions by the nurse is appropriate? a) Administer the IV calcium gluconate ordered. b) Administer the oral calcium supplement ordered. c) Administer the sedative ordered. d) Start administration of oxygen at 2 L/min per cannula.

A. When hypocalcemia and tetany occur after a thyroidectomy, the immediate treatment is administration of IV calcium gluconate. If this does not decrease neuromuscular irritability and seizure activity immediately, sedative agents such as pentobarbital may be administered.

While recording the health history of a patient who is scheduled for a thyroid test, the nurse is informed by the patient about an allergy to shellfish. What is the nurse's most appropriate response? a) Document the allergy and inform the physician. b) Inquire about frequent urination. c) Consult the institution's procedure manual. d) Palpate the thyroid gland.

A. When thyroid tests are scheduled, it is necessary to determine whether the patient is allergic to iodine (shellfish) and whether the patient has taken medications or agents that contain iodine, as these may alter the test results. This information should be documented in the patient's medical record and in the laboratory requisition.

During an assessment of a patient's functional health pattern, which question by the nurse directly addresses the patient's thyroid function? a) "Do you experience fatigue even if you have slept a long time?" b) "Can you describe the amount of stress in your life?" c) "Have you experienced any headaches or sinus problems?" d) "Do you have to get up at night to empty your bladder?"

A. With the diagnosis of hypothyroidism, extreme fatigue makes it difficult for the person to complete a full day's work or participate in usual activities.

Interventions such as promotion of nutrition, exercise, and stress reduction should be undertaken by the nurse for patients who have HIV infection, primarily because these interventions will A. improve immune function B. increase the patient's strength and ability to care for him- or herself. C. promote a feeling of well-being in the patient. D. prevent transmission of the virus to others.

A. improve immune function

HIV patients who are suffering from depression should be frequently assessed for: A. suicidal ideation. B. fear of death. C. physical decline. D. support systems.

A. suicidal ideation.

In which order will the nurse take these steps to prepare NPH 20 units and regular insulin 2 units using the same syringe? (Put a comma and a space between each answer choice [A, B, C, D, E]). a. Rotate NPH vial. b. Withdraw regular insulin. c. Withdraw 20 units of NPH. d. Inject 20 units of air into NPH vial. e. Inject 2 units of air into regular insulin vial.

ANS: A, D, E, B, C When mixing regular insulin with NPH, it is important to avoid contact between the regular insulin and the additives in the NPH that slow the onset, peak, and duration of activity in the longer-acting insulin.

Which question during the assessment of a patient who has diabetes will help the nurse identify autonomic neuropathy? A. "Do you feel bloated after eating?" B. "Have you seen any skin changes?" C. "Do you need to increase your insulin dosage when you are stressed?" D. "Have you noticed any painful new ulcerations or sores on your feet?"

ANS: A Autonomic neuropathy can cause delayed gastric emptying, which results in a bloated feeling for the patient. The other questions are also appropriate to ask but would not help in identifying autonomic neuropathy.

To assist an older patient with diabetes to engage in moderate daily exercise, which action is most important for the nurse to take? A. Determine what types of activities the patient enjoys. B. Remind the patient that exercise improves self-esteem. C. Teach the patient about the effects of exercise on glucose level. D. Give the patient a list of activities that are moderate in intensity.

ANS: A Because consistency with exercise is important, assessment for the types of exercise that the patient finds enjoyable is the most important action by the nurse in ensuring adherence to an exercise program. The other actions may be helpful but are not the most important in improving compliance.

Which nursing action can the nurse delegate to experienced unlicensed assistive personnel (UAP) who are working in the diabetic clinic? A. Measure the ankle-brachial index. B. Check for changes in skin pigmentation. C. Assess for unilateral or bilateral foot drop. D. Ask the patient about symptoms of depression.

ANS: A Checking systolic pressure at the ankle and brachial areas and calculating the ankle-brachial index is a procedure that can be done by UAP who have been trained in the procedure. The other assessments require more education and critical thinking and should be done by the registered nurse (RN).

A 27-yr-old patient admitted with diabetic ketoacidosis (DKA) has a serum glucose level of 732 mg/dL and serum potassium level of 3.1 mEq/L. Which action prescribed by the health care provider should the nurse take first? A. Place the patient on a cardiac monitor. B. Administer IV potassium supplements. C. Ask the patient about home insulin doses. D. Start an insulin infusion at 0.1 units/kg/hr.

ANS: A Hypokalemia can lead to potentially fatal dysrhythmias such as ventricular tachycardia and ventricular fibrillation, which would be detected with electrocardiogram (ECG) monitoring. Because potassium must be infused over at least 1 hour, the nurse should initiate cardiac monitoring before infusion of potassium. Insulin should not be administered without cardiac monitoring because insulin infusion will further decrease potassium levels. Discussion of home insulin and possible causes can wait until the patient is stabilized.

Which patient action indicates a good understanding of the nurse's teaching about the use of an insulin pump? A. The patient programs the pump for an insulin bolus after eating. B. The patient changes the location of the insertion site every week. C. The patient takes the pump off at bedtime and starts it again each morning. D. The patient plans a diet with more calories than usual when using the pump.

ANS: A In addition to the basal rate of insulin infusion, the patient will adjust the pump to administer a bolus after each meal, with the dosage depending on the oral intake. The insertion site should be changed every 2 or 3 days. There is more flexibility in diet and exercise when an insulin pump is used, but it does not provide for consuming a higher calorie diet. The pump will deliver a basal insulin rate 24 hours a day.

In assessing a patient with AIDS for risk factors, the nurse recognizes the most risky behavior that the patient reported as: A. Oral sex without contact with glans penis B. Oral sex with a condom C. Anal sex with condom D. Use of sex toys

C. Anal sex with condom

A female patient is scheduled for an oral glucose tolerance test. Which information from the patient's health history is important for the nurse to communicate to the health care provider regarding this test? A. The patient uses oral contraceptives. B. The patient runs several days a week. C. The patient has been pregnant three times. D. The patient has a family history of diabetes.

ANS: A Oral contraceptive use may falsely elevate oral glucose tolerance test (OGTT) values. Exercise and a family history of diabetes both can affect blood glucose but will not lead to misleading information from the OGTT. History of previous pregnancies may provide informational about gestational glucose tolerance but will not lead to misleading information from the OGTT.

A patient with diabetes is starting on intensive insulin therapy. Which type of insulin will the nurse discuss using for mealtime coverage? A. Lispro (Humalog) B. Detemir (Levemir) C. Glargine (Lantus) D. NPH (Humulin N)

ANS: A Rapid- or short-acting insulin is used for mealtime coverage for patients receiving intensive insulin therapy. NPH, glargine, or detemir will be used as the basal insulin.

Which statement by the patient indicates a need for additional instruction in administering insulin? A. "I need to rotate injection sites among my arms, legs, and abdomen each day." B. "I can buy the 0.5-mL syringes because the line markings will be easier to see." C. "I do not need to aspirate the plunger to check for blood before injecting insulin." D. "I should draw up the regular insulin first, after injecting air into the NPH bottle."

ANS: A Rotating sites is no longer recommended because there is more consistent insulin absorption when the same site is used consistently. The other patient statements are accurate and indicate that no additional instruction is needed.

The nurse has administered 4 oz of orange juice to an alert patient whose blood glucose was 62 mg/dL. Fifteen minutes later, the blood glucose is 67 mg/dL. Which action should the nurse take next? a. Give the patient 4 to 6 oz more orange juice. b. Administer the PRN glucagon (Glucagon) 1 mg IM. c. Have the patient eat some peanut butter with crackers. d. Notify the health care provider about the hypoglycemia.

ANS: A The "rule of 15" indicates that administration of quickly acting carbohydrates should be done two or three times for a conscious patient whose glucose remains less than 70 mg/dL before notifying the health care provider. More complex carbohydrates and fats may be used after the glucose has stabilized. Glucagon should be used if the patient's level of consciousness decreases so that oral carbohydrates can no longer be given.

A patient who has type 1 diabetes plans to swim laps for an hour daily at 1:00 PM. The clinic nurse will plan to teach the patient to A. check glucose level before, during, and after swimming. B. delay eating the noon meal until after the swimming class. C. increase the morning dose of neutral protamine Hagedorn (NPH) insulin. D. time the morning insulin injection so that the peak occurs while swimming.

ANS: A The change in exercise will affect blood glucose, and the patient will need to monitor glucose carefully to determine the need for changes in diet and insulin administration. Because exercise tends to decrease blood glucose, patients are advised to eat before exercising. Increasing the morning NPH or timing the insulin to peak during exercise may lead to hypoglycemia, especially with the increased exercise.

The nurse is assessing a 55-yr-old female patient with type 2 diabetes who has a body mass index (BMI) of 31 kg/m2. Which goal in the plan of care is most important for this patient? A. The patient will reach a glycosylated hemoglobin level of less than 7%. B. The patient will follow a diet and exercise plan that results in weight loss. C. The patient will choose a diet that distributes calories throughout the day. D. The patient will state the reasons for eliminating simple sugars in the diet.

ANS: A The complications of diabetes are related to elevated blood glucose and the most important patient outcome is the reduction of glucose to near-normal levels. A BMI of 30?9?kg/m2 or above is considered obese, so the other outcomes are appropriate but are not as high in priority.

A patient with diabetic ketoacidosis is brought to the emergency department. Which prescribed action should the nurse implement first? A. Infuse 1 L of normal saline per hour. B. Give sodium bicarbonate 50 mEq IV push. C. Administer regular insulin 10 U by IV push. D. Start a regular insulin infusion at 0.1 units/kg/hr.

ANS: A The most urgent patient problem is the hypovolemia associated with diabetic ketoacidosis (DKA), and the priority is to infuse IV fluids. The other actions can be done after the infusion of normal saline is initiated.

Which information will the nurse include in teaching a female patient who has peripheral arterial disease, type 2 diabetes, and sensory neuropathy of the feet and legs? A. Choose flat-soled leather shoes. B. Set heating pads on a low temperature. C. Use callus remover for corns or calluses. D. Soak feet in warm water for an hour each day.

ANS: A The patient is taught to avoid high heels and that leather shoes are preferred. The feet should be washed, but not soaked, in warm water daily. Heating pad use should be avoided. Commercial callus and corn removers should be avoided. The patient should see a specialist to treat these problems.

A patient receives aspart (NovoLog) insulin at 0800. At which time would the nurse anticipate the highest risk for hypoglycemia? A. 1000 B. 1400 C. 12:00 AM D. 1600

ANS: A The rapid-acting insulins peak in 1 to 3 hours. The patient is not at a high risk for hypoglycemia at the other listed times, although hypoglycemia may occur.

After change-of-shift report, which patient should the nurse assess first? A. A 19-yr-old patient with type 1 diabetes who has a hemoglobin A1C of 12% B. A 23-yr-old patient with type 1 diabetes who has a blood glucose of 40 mg/dL C. A 40-yr-old patient who is pregnant and whose oral glucose tolerance test is 202 mg/dL D. A 50-yr-old patient who uses exenatide (Byetta) and is complaining of acute abdominal pain

ANS: B Because the brain requires glucose to function, untreated hypoglycemia can cause unconsciousness, seizures, and death. The nurse will rapidly assess and treat the patient with low blood glucose. The other patients also have symptoms that require assessments or interventions, but they are not at immediate risk for life-threatening complications.

A client is admitted to the hospital after being treated with vincristine for cancer. The client reports being diagnosed with peripheral neuropathy due to chemotherapy. When assessing the client, the nurse should expect to see which of the following clinical manifestations in the client's extremities? a. edema b. tingling c. ataxia d. spasms

B Several chemotherapeutic agents, including vincristine, may cause peripheral neuropathy. One of the major manifestations of peripheral neuropathy is numbness and/or tingling of an extremity.

The nurse is preparing to teach a 43-yr-old man who is newly diagnosed with type 2 diabetes about home management of the disease. Which action should the nurse take first? A. Ask the patient's family to participate in the diabetes education program. B. Assess the patient's perception of what it means to have diabetes mellitus. C. Demonstrate how to check glucose using capillary blood glucose monitoring. D. Discuss the need for the patient to actively participate in diabetes management.

ANS: B Before planning teaching, the nurse should assess the patient's interest in and ability to self-manage the diabetes. After assessing the patient, the other nursing actions may be appropriate, but planning needs to be individualized to each patient.

Which patient action indicates good understanding of the nurse's teaching about administration of aspart (NovoLog) insulin? A. The patient avoids injecting the insulin into the upper abdominal area. B. The patient cleans the skin with soap and water before insulin administration. C. The patient stores the insulin in the freezer after administering the prescribed dose. D. The patient pushes the plunger down while removing the syringe from the injection site.

ANS: B Cleaning the skin with soap and water is acceptable. Insulin should not be frozen. The patient should leave the syringe in place for about 5 seconds after injection to be sure that all the insulin has been injected. The upper abdominal area is one of the preferred areas for insulin injection.

The nurse determines a need for additional instruction when the patient with newly diagnosed type 1 diabetes says which of the following? A. "I will need a bedtime snack because I take an evening dose of NPH insulin." B. "I can choose any foods, as long as I use enough insulin to cover the calories." C. "I can have an occasional beverage with alcohol if I include it in my meal plan." D. "I will eat something at meal times to prevent hypoglycemia, even if I am not hungry."

ANS: B Most patients with type 1 diabetes need to plan diet choices very carefully. Patients who are using intensified insulin therapy have considerable flexibility in diet choices but still should restrict dietary intake of items such as fat, protein, and alcohol. The other patient statements are correct and indicate good understanding of the diet instruction.

The nurse is taking a health history from a 29-yr-old pregnant patient at the first prenatal visit. The patient reports that she has no personal history of diabetes, but her mother has diabetes. Which action will the nurse plan to take? A. Teach the patient about administering regular insulin. B. Schedule the patient for a fasting blood glucose level. C. Teach about an increased risk for fetal problems with gestational diabetes. D. Schedule an oral glucose tolerance test for the twenty-fourth week of pregnancy.

ANS: B Patients at high risk for gestational diabetes should be screened for diabetes on the initial prenatal visit. An oral glucose tolerance test may also be used to check for diabetes, but it would be done before the twenty-fourth week. Teaching plans would depend on the outcome of a fasting blood glucose test and other tests.

A patient with diabetes rides a bicycle to and from work every day. Which site should the nurse teach the patient to use to administer the morning insulin? A. thigh. B. abdomen. C. buttock. D. upper arm.

ANS: B Patients should be taught not to administer insulin into a site that will be exercised because exercise will increase the rate of absorption. The thigh, buttock, and arm are all exercised by riding a bicycle.

Which action should the nurse take after a patient treated with intramuscular glucagon for hypoglycemia regains consciousness? A. Assess the patient for symptoms of hyperglycemia. B. Give the patient a snack of peanut butter and crackers. C. Have the patient drink a glass of orange juice or nonfat milk. D. Administer a continuous infusion of 5% dextrose for 24 hours.

ANS: B Rebound hypoglycemia can occur after glucagon administration, but having a meal containing complex carbohydrates plus protein and fat will help prevent hypoglycemia. Orange juice and nonfat milk will elevate blood glucose rapidly, but the cheese and crackers will stabilize blood glucose. Administration of IV glucose might be used in patients who were unable to take in nutrition orally. The patient should be assessed for symptoms of hypoglycemia after glucagon administration.

A patient with type 2 diabetes is scheduled for a follow-up visit in the clinic several months from now. Which test will the nurse schedule to evaluate the effectiveness of treatment for the patient? A. Fasting blood glucose B. Glycosylated hemoglobin C. Oral glucose tolerance D. Urine dipstick for glucose

ANS: B The glycosylated hemoglobin (A1C) test shows the overall control of glucose over 90 to 120 days. A fasting blood level indicates only the glucose level at one time. Urine glucose testing is not an accurate reflection of blood glucose level and does not reflect the glucose over a prolonged time. Oral glucose tolerance testing is done to diagnose diabetes but is not used for monitoring glucose control after diabetes has been diagnosed.

Which laboratory value reported to the nurse by the unlicensed assistive personnel (UAP) indicates an urgent need for the nurse's assessment of the patient? A. Bedtime glucose of 140 mg/dL B. Noon blood glucose of 52 mg/dL C. Fasting blood glucose of 130 mg/dL D. 2-hr postprandial glucose of 220 mg/dL

ANS: B The nurse should assess the patient with a blood glucose level of 52 mg/dL for symptoms of hypoglycemia and give the patient a carbohydrate-containing beverage such as orange juice. The other values are within an acceptable range or not immediately dangerous for a patient with diabetes.

The nurse identifies a need for additional teaching when the patient who is self-monitoring blood glucose A. washes the puncture site using warm water and soap. B. chooses a puncture site in the center of the finger pad. C. hangs the arm down for a minute before puncturing the site. D. says the result of 120 mg indicates good blood sugar control.

ANS: B The patient is taught to choose a puncture site at the side of the finger pad because there are fewer nerve endings along the side of the finger pad. The other patient actions indicate that teaching has been effective.

Which information will the nurse include when teaching a patient who has type 2 diabetes about glyburide ? A. Glyburide decreases glucagon secretion from the pancreas. B. Glyburide stimulates insulin production and release from the pancreas. C. Glyburide should be taken even if the morning blood glucose level is low. D. Glyburide should not be used for 48 hours after receiving IV contrast media.

ANS: B The sulfonylureas stimulate the production and release of insulin from the pancreas. If the glucose level is low, the patient should contact the health care provider before taking glyburide because hypoglycemia can occur with this class of medication. Metformin should be held for 48 hours after administration of IV contrast media, but this is not necessary for glyburide. Glucagon secretion is not affected by glyburide.

An active 32-yr-old male who has type 1 diabetes is being seen in the endocrine clinic. Which finding indicates a need for the nurse to discuss a possible a change in therapy with the health care provider? A. Hemoglobin A1C level of 6.2% B. Blood pressure of 140/88 mmHg C. Heart rate at rest of 58 beats/minute D. High density lipoprotein (HDL) level of 65 mg/dL

ANS: B To decrease the incidence of macrovascular and microvascular problems in patients with diabetes, the goal blood pressure is usually 130/80 mm Hg. An A1C less than 6.5%, a low resting heart rate (consistent with regular aerobic exercise in a young adult), and an HDL level of 65 mg/dL all indicate that the patient's diabetes and risk factors for vascular disease are well controlled.

A patient who has diabetes and reported burning foot pain at night receives a new prescription. Which information should the nurse teach the patient about amitriptyline ? A. Amitriptyline decreases the depression caused by your foot pain. B. Amitriptyline helps prevent transmission of pain impulses to the brain. C. Amitriptyline corrects some of the blood vessel changes that cause pain. D. Amitriptyline improves sleep and makes you less aware of nighttime pain.

ANS: B Tricyclic antidepressants (TCAs) decrease the transmission of pain impulses to the spinal cord and brain. TCAs also improve sleep quality and are used for depression, but that is not the major purpose for their use in diabetic neuropathy. The blood vessel changes that contribute to neuropathy are not affected by TCAs.

The nurse is assessing a 22-yr-old patient experiencing the onset of symptoms of type 1 diabetes. To which question would the nurse anticipate a positive response? A. "Are you anorexic?" B. "Have you lost weight lately?" C. "Is your urine dark colored?" D. "Do you crave sugary drinks?"

ANS: B Weight loss occurs because the body is no longer able to absorb glucose and starts to break down protein and fat for energy. The patient is thirsty but does not necessarily crave sugar-containing fluids. Increased appetite is a classic symptom of type 1 diabetes. With the classic symptom of polyuria, urine will be very dilute.

1. To monitor for complications in a patient with type 2 diabetes, which tests will the nurse in the diabetic clinic schedule at least annually (select all that apply)? A. Chest x-ray B. Blood pressure C. Serum creatinine D. Urine for microalbuminuria E. Complete blood count (CBC) F. Monofilament testing of the foot

ANS: B, C, D, F Blood pressure, serum creatinine, urine testing for microalbuminuria, and monofilament testing of the foot are recommended at least annually to screen for possible microvascular and macrovascular complications of diabetes. Chest x-ray and CBC might be ordered if the patient with diabetes presents with symptoms of respiratory or infectious problems but are not routinely included in screening.

After the nurse has finished teaching a patient who has a new prescription for exenatide (Byetta), which patient statement indicates that the teaching has been effective? A. "I may feel hungrier than usual when I take this medicine." B. "I will not need to worry about hypoglycemia with the Byetta." C. "I should take my daily aspirin at least an hour before the Byetta." D. "I will take the pill at the same time I eat breakfast in the morning."

ANS: C Because exenatide slows gastric emptying, oral medications should be taken at least 1 hour before the exenatide to avoid slowing absorption. Exenatide is injected and increases feelings of satiety. Hypoglycemia can occur with this medication.

A 30-yr-old patient has a new diagnosis of type 2 diabetes. The nurse will discuss the need to schedule a dilated eye examination A. every 2 years. B. when the patient is 39 years old. C. as soon as possible. D. within the first year after diagnosis.

ANS: C Because many patients have some diabetic retinopathy when they are first diagnosed with type 2 diabetes, a dilated eye examination is recommended at the time of diagnosis and annually thereafter. Patients with type 1 diabetes should have dilated eye examinations starting 5 years after they are diagnosed and then annually.

Which statement by a nurse to a patient newly diagnosed with type 2 diabetes is correct? A.Insulin is not used to control blood glucose in patients with type 2 diabetes. B. Complications of type 2 diabetes are less serious than those of type 1 diabetes. C. Changes in diet and exercise may control blood glucose levels in type 2 diabetes. D. Type 2 diabetes is usually diagnosed when the patient is admitted with a hyperglycemic coma.

ANS: C For some patients with type 2 diabetes, changes in lifestyle are sufficient to achieve blood glucose control. Insulin is frequently used for type 2 diabetes, complications are equally severe as for type 1 diabetes, and type 2 diabetes is usually diagnosed with routine laboratory testing or after a patient develops complications such as frequent yeast infections.

When a patient who takes metformin (Glucophage) to manage type 2 diabetes develops an allergic rash from an unknown cause, the health care provider prescribes prednisone. The nurse will anticipate that the patient may A. need a diet higher in calories while receiving prednisone. B. develop acute hypoglycemia while taking the prednisone. C. require administration of insulin while taking prednisone. D. have rashes caused by metformin-prednisone interactions.

ANS: C Glucose levels increase when patients are taking corticosteroids, and insulin may be required to control blood glucose. Hypoglycemia is not a side effect of prednisone. Rashes are not an adverse effect caused by taking metformin and prednisone simultaneously. The patient may have an increased appetite when taking prednisone but will not need a diet that is higher in calories.

An unresponsive patient with type 2 diabetes is brought to the emergency department and diagnosed with hyperosmolar hyperglycemic syndrome (HHS). The nurse will anticipate the need to A. give 50% dextrose. B. initiate O2 by nasal cannula. C. insert an IV catheter. D. administer glargine (Lantus) insulin.

ANS: C HHS is initially treated with large volumes of IV fluids to correct hypovolemia. Regular insulin is administered, not a long-acting insulin. There is no indication that the patient requires O2. Dextrose solutions will increase the patient's blood glucose and would be contraindicated.

The health care provider suspects the Somogyi effect in a 50-yr-old patient whose 6:00 AM blood glucose is 230 mg/dL. Which action will the nurse teach the patient to take? A. Avoid snacking at bedtime. B. Increase the rapid-acting insulin dose. C. Check the blood glucose during the night D. Administer a larger dose of long-acting insulin.

ANS: C If the Somogyi effect is causing the patient's increased morning glucose level, the patient will experience hypoglycemia between 2:00 and 4:00 AM. The dose of insulin will be reduced, rather than increased. A bedtime snack is used to prevent hypoglycemic episodes during the night.

A 26-yr-old female with type 1 diabetes develops a sore throat and runny nose after caring for her sick toddler. The patient calls the clinic for advice about her symptoms and a blood glucose level of 210 mg/dL despite taking her usual glargine (Lantus) and lispro (Humalog) insulin. The nurse advises the patient to A. use only the lispro insulin until the symptoms are resolved. B. limit intake of calories until the glucose is less than 120 mg/dL. C. monitor blood glucose every 4 hours and notify the clinic if it continues to rise. D. decrease intake of carbohydrates until glycosylated hemoglobin is less than 7%.

ANS: C Infection and other stressors increase blood glucose levels and the patient will need to test blood glucose frequently, treat elevations appropriately with lispro insulin, and call the health care provider if glucose levels continue to be elevated. Discontinuing the glargine will contribute to hyperglycemia and may lead to diabetic ketoacidosis (DKA). Decreasing carbohydrate or caloric intake is not appropriate because the patient will need more calories when ill. Glycosylated hemoglobin testing is not used to evaluate short-term alterations in blood glucose.

Which action by a patient indicates that the home health nurse's teaching about glargine and regular insulin has been successful? A. The patient administers the glargine 30 minutes before each meal. B. The patient's family prefills the syringes with the mix of insulins weekly. C. The patient discards the open vials of glargine and regular insulin after 4 weeks. D. The patient draws up the regular insulin and then the glargine in the same syringe.

ANS: C Insulin can be stored at room temperature for 4 weeks. Glargine should not be mixed with other insulins or prefilled and stored. Short-acting regular insulin is administered before meals, and glargine is given once daily.

When a patient with type 2 diabetes is admitted for a cholecystectomy, which nursing action can the nurse delegate to a licensed practical/vocational nurse (LPN/LVN)? A. Communicate the blood glucose level and insulin dose to the circulating nurse in surgery. B. Discuss the reason for the use of insulin therapy during the immediate postoperative period. C. Administer the prescribed lispro (Humalog) insulin before transporting the patient to surgery. D. Plan strategies to minimize the risk for hypoglycemia or hyperglycemia during the postoperative period.

ANS: C LPN/LVN education and scope of practice includes administration of insulin. Communication about patient status with other departments, planning, and patient teaching are skills that require RN education and scope of practice.

A few weeks after an 82-yr-old patient with a new diagnosis of type 2 diabetes has been placed on metformin (Glucophage) therapy and taught about appropriate diet and exercise, the home health nurse makes a visit. Which finding should the nurse promptly discuss with the health care provider? A. Hemoglobin A1C level is 7.9%. B. Last eye examination was 18 months ago. C. Glomerular filtration rate is decreased. D. Patient has questions about the prescribed diet.

ANS: C The decrease in renal function may indicate a need to adjust the dose of metformin or change to a different medication. In older patients, the goal for A1C may be higher in order to avoid complications associated with hypoglycemia. The nurse will plan on scheduling the patient for an eye examination and addressing the questions about diet, but the area for prompt intervention is the patient's decreased renal function.

A patient who was admitted with diabetic ketoacidosis secondary to a urinary tract infection has been weaned off an insulin drip 30 minutes ago. The patient reports feeling lightheaded and sweaty. Which action should the nurse take first? A. Infuse dextrose 50% by slow IV push. B. Administer 1 mg glucagon subcutaneously. C. Obtain a glucose reading using a finger stick. D. Have the patient drink 4 ounces of orange juice.

ANS: C The patient's clinical manifestations are consistent with hypoglycemia, and the initial action should be to check the patient's glucose with a finger stick or order a stat blood glucose. If the glucose is low, the patient should ingest a rapid-acting carbohydrate, such as orange juice. Glucagon or dextrose 50% might be given if the patient's symptoms become worse or if the patient is unconscious.

After change-of-shift report, which patient will the nurse assess first? A. A 19-yr-old patient with type 1 diabetes who was admitted with possible dawn phenomenon B. A 35-yr-old patient with type 1 diabetes whose most recent blood glucose reading was 230 mg/dL C. A 60-yr-old patient with hyperosmolar hyperglycemic syndrome who has poor skin turgor and dry oral mucosa D. A 68-yr-old patient with type 2 diabetes who has severe peripheral neuropathy and complains of burning foot pain

ANS: C The patient's diagnosis of HHS and signs of dehydration indicate that the nurse should rapidly assess for signs of shock and determine whether increased fluid infusion is needed. The other patients also need assessment and intervention but do not have life-threatening complications.

A patient screened for diabetes at a clinic has a fasting plasma glucose level of 120 mg/dL (6.7 mmol/L). The nurse will plan to teach the patient about A. self-monitoring of blood glucose. B. using low doses of regular insulin. C. lifestyle changes to lower blood glucose. D. effects of oral hypoglycemic medications

ANS: C The patient's impaired fasting glucose indicates prediabetes, and the patient should be counseled about lifestyle changes to prevent the development of type 2 diabetes. The patient with prediabetes does not require insulin or oral hypoglycemics for glucose control and does not need to self-monitor blood glucose.

A patient who has type 2 diabetes is being prepared for an elective coronary angiogram. Which information would the nurse anticipate might lead to rescheduling the test? A. The patient's most recent A1C was 6.5%. B. The patient's blood glucose is 128 mg/dL. C. The patient took the prescribed metformin today. D. The patient took the prescribed captopril this morning.

ANS: C To avoid lactic acidosis, metformin should be discontinued a day or 2 before the coronary angiogram and should not be used for 48 hours after IV contrast media are administered. The other patient data will also be reported but do not indicate any need to reschedule the procedure.

A hospitalized diabetic patient received 38 U of NPH insulin at 7:00 AM. At 1:00 PM, the patient has been away from the nursing unit for 2 hours, missing the lunch delivery while awaiting a chest x-ray. To prevent hypoglycemia, the best action by the nurse is to A. save the lunch tray for the patient's later return to the unit. B. ask that diagnostic testing area staff to start a 5% dextrose IV. C. send a glass of milk or orange juice to the patient in the diagnostic testing area. D. request that if testing is further delayed, the patient be returned to the unit to eat.

ANS: D Consistency for mealtimes assists with regulation of blood glucose, so the best option is for the patient to have lunch at the usual time. Waiting to eat until after the procedure is likely to cause hypoglycemia. Administration of an IV solution is unnecessarily invasive for the patient. A glass of milk or juice will keep the patient from becoming hypoglycemic but will cause a rapid rise in blood glucose because of the rapid absorption of the simple carbohydrate in these items.

The nurse is interviewing a new patient with diabetes who takes rosiglitazone (Avandia). Which information would the nurse anticipate resulting in the health care provider discontinuing the medication? A. The patient's blood pressure is 154/92. B. The patient's blood glucose is 86 mg/dL. C. The patient reports a history of emphysema. D. The patient has chest pressure when walking.

ANS: D Rosiglitazone can cause myocardial ischemia. The nurse should immediately notify the health care provider and expect orders to discontinue the medication. A blood glucose level of 86 mg/dL indicates a positive effect from the medication. Hypertension and a history of emphysema do not contraindicate this medication.

Which finding indicates a need to contact the health care provider before the nurse administers metformin (Glucophage)? A. The patient's blood glucose level is 174 mg/dL. B. The patient is scheduled for a chest x-ray in an hour. C. The patient has gained 2 lb (0.9 kg) in the past 24 hours. D.The patient's blood urea nitrogen (BUN) level is 52 mg/dL.

ANS: D The BUN indicates possible renal failure, and metformin should not be used in patients with renal failure. The other findings are not contraindications to the use of metformin.

The nurse has been teaching a patient with type 2 diabetes about managing blood glucose levels and taking glipizide (Glucotrol). Which patient statement indicates a need for additional teaching? A. "If I overeat at a meal, I will still take the usual dose of medication." B. "Other medications besides the Glucotrol may affect my blood sugar." C. "When I am ill, I may have to take insulin to control my blood sugar." D. "My diabetes won't cause complications because I don't need insulin."

ANS: D The patient should understand that type 2 diabetes places the patient at risk for many complications and that good glucose control is as important when taking oral agents as when using insulin. The other statements are accurate and indicate good understanding of the use of glipizide.

A 28-yr-old male patient with type 1 diabetes reports how he manages his exercise and glucose control. Which behavior indicates that the nurse should implement additional teaching? A. The patient always carries hard candies when engaging in exercise. B. The patient goes for a vigorous walk when his glucose is 200 mg/dL. C. The patient has a peanut butter sandwich before going for a bicycle ride. D. The patient increases daily exercise when ketones are present in the urine.

ANS: D When the patient is ketotic, exercise may result in an increase in blood glucose level. Patients with type 1 diabetes should be taught to avoid exercise when ketosis is present. The other statements are correct.

Nutritional Therapy

According to the ADA, guidlines indicate that the overall healthy eating plan, a person with DM can eat the same foods as a person who does not have diabetes. The same principles of good nutrition that apply to the general population also apply to the person with diabetes. TYPE I DIABETES = meal planning should be based on the individual's usual food intake and balanced with insulin and exercise patterns. TYPE II DIABETES = should emphasize achieving glucose, lipid, and BP goals. Modest weight loss has been associated with improved insulin resistance and glycemic control. Important to consult with the dietitian.

List the inhaled insulin

Afrezza

A client tells the oncology nurse about an upcoming vacation to the beach to celebrate completing radiation treatments for cancer. What response by the nurse is most appropriate? a. Avoid getting salt water on the radiation site. b. Do not expose the radiation area to direct sunlight. c. Have a wonderful time and enjoy your vacation! d. Remember you should not drink alcohol for a year.

B The skin overlying the radiation site is extremely sensitive to sunlight after radiation therapy has been completed. The nurse should inform the client to avoid sun exposure to this area. This advice continues for 1 year after treatment has been completed. The other statements are not appropriate.

Hypoglycemia

At the FIRST sign of hypoglycemia, check the blood glucose if possible. If it is <70, immediately begin treatment of hypoglycemia. If the patient has manifestations of hypoglycemia and monitoring equipment is not available or the patient has a history of chronic poor glycemic control, hypoglycemia should be assumed and treatment initiated. -Hypoglycemia is treated by ingesting 15 g of a SIMPLE sugar (fast acting) carbohydrate, such as 4-6 oz of fruit juice or regular soft drink. Avoid overtreatment with large quantities of quick acting carbohydrates, such as candy bars, so that a rapid fluctuation to hyperglycemia does not occur. -Recheck the blood glucose 15 minutes later. If the value is still below 70, have the patient ingest 15 g more of carbohydrate and recheck the blood glucose in 15 minutes. Because of the potential for rebound hypoglycemia after an acute episode, have the patient ingest a complex carbohydrate after recovery to prevent another hypoglycemic attack. -If no significant improvement occurs after 2-3 doses of 15 g of simple carbohydrate, contact the health care provider. Once acute hypoglycemia has been reversed, you should explore with the patient the reasons why the situation developed. This may include additional teaching of the patient or caregiver to avoid future episodes.

A 16-year-old client newly diagnosed with type 1 diabetes has a very low body weight despite eating regular meals. The client is upset because friends frequently state, "You look anorexic." Which statement by the nurse would be the best response to help this client understand the cause of weight loss due to this condition? A. "You may be having undiagnosed infections, causing you to lose extra weight." B. "Your body is using protein and fat for energy instead of glucose." C. "I will refer you to a dietician who can help you with your weight." D. "Don't worry about what your friends think; the carbohydrates you eat are being quickly digested, increasing your metabolism."

B

The client who is managing diabetes through diet and insulin control asks the nurse why exercise is important. Which is the best response by the nurse to support adding exercise to the daily routine? A. Decreases risk of developing insulin resistance and hyperglycemia B. Increases ability for glucose to get into the cell and lowers blood sugar C Creates an overall feeling of well-being and lowers risk of depression D. Decreases need for pancreas to produce more cells

B

The nurse is preparing a presentation for a group of adults at a local community center about diabetes. Which of the following would the nurse include as associated with type 2 diabetes? A. Little relation to prediabetes B. Insufficient insulin production C. . Onset most common during adolescence D. Less common than type 1 diabetes

B

Which combination of adverse effects should a nurse monitor for when administering IV insulin to a client with diabetic ketoacidosis? A. Hypocalcemia and hyperkalemia B. Hypokalemia and hypoglycemia C. Hypernatremia and hypercalcemia D. Hyperkalemia and hyperglycemia

B

Which instruction about insulin administration should a nurse give to a client? A. "Discard the intermediate-acting insulin if it appears cloudy." B. "Always follow the same order when drawing the different insulins into the syringe." C. "Shake the vials before withdrawing the insulin." D. "Store unopened vials of insulin in the freezer at temperatures well below freezing."

B

Which of the following factors would a nurse identify as a most likely cause of diabetic ketoacidosis (DKA) in a client with diabetes? A.The client continues medication therapy despite adequate food intake. B. The client has eaten and has not taken or received insulin. C. The client has not consumed sufficient calories. D. The client has been exercising more than usual.

B

A client with diabetes mellitus has a prescription for 5 units of U-100 regular insulin and 25 units of U-100 isophane insulin suspension (NPH) to be taken before breakfast. At about 4:30 p.m., the client experiences headache, sweating, tremor, pallor, and nervousness. What is the most probable cause of these signs and symptoms? A. Serum glucose level of 450 mg/dl B. Serum glucose level of 52 mg/dl C. Serum calcium level of 8.9 mg/dl D. Serum calcium level of 10.2 mg/dl

B Headache, sweating, tremor, pallor, and nervousness typically result from hypoglycemia, an insulin reaction in which serum glucose level drops below 70 mg/dl. Hypoglycemia may occur 4 to 18 hours after administration of isophane insulin suspension or insulin zinc suspension (Lente), which are intermediate-acting insulins. Although hypoglycemia may occur at any time, it usually precedes meals. Hyperglycemia, in which serum glucose level is above 180 mg/dl, causes such early manifestations as fatigue, malaise, drowsiness, polyuria, and polydipsia.

A client has a platelet count of 9800/mm3. What action by the nurse is most appropriate? a. Assess the client for calf pain, warmth, and redness. b. Instruct the client to call for help to get out of bed. c. Obtain cultures as per the facilitys standing policy. d. Place the client on protective isolation precautions.

B A client with a platelet count this low is at high risk for serious bleeding episodes. To prevent injury, the client should be instructed to call for help prior to getting out of bed. Calf pain, warmth, and redness might indicate a deep vein thrombosis, not associated with low platelets. Cultures and isolation relate to low white cell counts.

A client with cancer has anorexia and mucositis, and is losing weight. The clients family members continually bring favorite foods to the client and are distressed when the client wont eat them. What action by the nurse is best? a. Explain the pathophysiologic reasons behind the client not eating. b. Help the family show other ways to demonstrate love and caring. c. Suggest foods and liquids the client might be willing to try to eat. d. Tell the family the client isn't able to eat now no matter what they bring.

B Families often become distressed when their loved ones wont eat. Providing food is a universal sign of caring, and to some people the refusal to eat signifies worsening of the condition. The best option for the nurse is to help the family find other ways to demonstrate caring and love, because with treatment-related anorexia and mucositis, the client is not likely to eat anything right now. Explaining the rationale for the problem is a good idea but does not suggest to the family anything that they can do for the client. Simply telling the family the client is not able to eat does not give them useful information and is dismissive of their concerns.

A client is being evaluated in an oncology clinic after referral by the client's provider for suspicion of Hodgkin's disease. Secondary to this possible diagnosis, the nurse should focus on assessing for which of the following? a. bone and joint pain b. enlarged lymph nodes c. difficulty swallowing d. patchy alopecia

B Hodgkin's disease is a malignancy of lymphoid tissue found in the lymph nodes, spleen, liver, and bone marrow. The first sign of cancer is often an enlarged lymph node which appears without a known cause. The disease can spread to adjacent lymph nodes and later may spread outside the lymph nodes to the lungs, liver, bones, or bone marrow

A client is concerned about the risk of lymphedema after a mastectomy. Which response by the nurse is best? a. You do not need to worry about lymphedema since you did not have radiation therapy. b. A risk factor for lymphedema is infection, so wear gloves when gardening outside. c. Numbness, tingling, and swelling are common sensations after a mastectomy. d. The risk for lymphedema is a real threat and can be very self-limiting.

B Infection can create lymphedema; therefore, the client needs to be cautious with activities using the affected arm, such as gardening. Radiation therapy is just one of the factors that could cause lymphedema. Other risk factors include obesity and the presence of axillary disease. The symptoms of lymphedema are heaviness, aching, fatigue, numbness, tingling, and swelling, and are not common after the surgery. Women with lymphedema live fulfilling lives.

A nurse in an oncology clinic is conducting an assessment on a client with multiple myeloma. In relation to lab findings commonly associated with this diagnosis, the nurse should carefully assess the client for manifestations related to which of the following? a. hypocalcemia b. thrombocytopenia c. leukocytosis d. polycythemia

B Multiple myeloma is the excessive growth and proliferation of abnormal plasma cells in the lymph nodes and bone marrow. The growth of these extra plasma cells interferes with the production of red blood cells (causing anemia), white blood cells (causing increased susceptibility to infection), and platelets, or thrombocytes (causing bleeding tendencies). Other effects include bone pain, destruction, and renal failure.

After receiving the hand-off report, which client should the oncology nurse see first? a. Client who is afebrile with a heart rate of 108 beats/min b. Older client on chemotherapy with mental status changes c. Client who is neutropenic and in protective isolation d. Client scheduled for radiation therapy today

B Older clients often do not exhibit classic signs of infection, and often mental status changes are the first observation. Clients on chemotherapy who become neutropenic also often do not exhibit classic signs of infection. The nurse should assess the older client first. The other clients can be seen afterward.

A client has received a dose of ondansetron (Zofran) for nausea. What action by the nurse is most important? a. Assess the client for a headache. b. Assist the client in getting out of bed. c. Instruct the client to reduce salt intake. d. Weigh the client daily before the client eats.

B Ondansetron side effects include postural hypotension, vertigo, and bradycardia, all of which increase the clients risk for injury. The nurse should assist the client when getting out of bed. Headache and fluid retention are not side effects of this drug.

A client in the oncology clinic reports her family is frustrated at her ongoing fatigue 4 months after radiation therapy for breast cancer. What response by the nurse is most appropriate? a. Are you getting adequate rest and sleep each day? b. It is normal to be fatigued even for years afterward. c. This is not normal and I will let the provider know. d. Try adding more vitamins B and C to your diet.

B Regardless of the cause, radiation-induced fatigue can be debilitating and may last for months or years after treatment has ended. Rest and adequate nutrition can affect fatigue, but it is most important that the client understands this is normal.

A client with breast cancer is receiving a combination of chemotherapy agents. The nurse identifies the rationale for this combined treatment modality is that the a. risk for renal toxicity is lessened b. agents act at different stages of cellular mitosis c. length of treatment will be shorter d. potential for bone marrow suppression is eliminated

B Different chemotherapeutic agents act at various stages of cellular mitosis (division). By combining agents, medication therapy is more effective in stopping or slowing the growth of cancerous cells by interfering with their ability to multiply

A client with uterine cancer is receiving chemotherapy and develops thrombocytopenia. Due to this side effect, the nurse should plan to give the greatest consideration to which of the following? a. monitoring visitors for signs of infection b. reminding the client to use an electric razor c. encouraging frequent rest periods d. instructing the client to vigorously brush and floss the teeth

B Thrombocytopenia is a decrease in the client's platelet count, which places the client at an increased risk of bleeding due to an inability to clot. Therefore, nursing measures that include taking bleeding precautions should be instituted, which include the use of electric, rather than blade, razors when shaving the face, underarms, and legs.

The student nurse caring for clients who have cancer understands that the general consequences of cancer include which client problems? (Select all that apply.) a. Clotting abnormalities from thrombocythemia b. Increased risk of infection from white blood cell deficits c. Nutritional deficits such as early satiety and cachexia d. Potential for reduced gas exchange e. Various motor and sensory deficits

B, C, D, E The general consequences of cancer include reduced immunity and blood-producing functions, altered GI structure and function, decreased respiratory function, and motor and sensory deficits. Clotting problems often occur due to thrombocytopenia (not enough platelets), not thrombocythemia (too many platelets).

A nurse is participating in primary prevention efforts directed against cancer. In which activities is this nurse most likely to engage? (Select all that apply.) a. Demonstrating breast self-examination methods to women b. Instructing people on the use of chemoprevention c. Providing vaccinations against certain cancers d. Screening teenage girls for cervical cancer e. Teaching teens the dangers of tanning booths

B, C, E Primary prevention aims to prevent the occurrence of a disease or disorder, in this case cancer. Secondary prevention includes screening and early diagnosis. Primary prevention activities include teaching people about chemoprevention, providing approved vaccinations to prevent cancer, and teaching teens the dangers of tanning beds. Breast examinations and screening for cervical cancer are secondary prevention methods.

The nurse would suspect an infection by CMV when the patient with AIDS says: A. "I need to get smaller clothes. I have lost 10 pounds in the last 6 weeks." B. "I need to get glasses, I can't see as well as I did a few months ago." C. "I need to take some pep pills. I don't have any energy." D."I need to drink more water. This diarrhea has really dehydrated me."

B. "I need to get glasses, I can't see as well as I did a few months ago."

The nurse explains that nucleoside reverse transcriptase inhibitor drugs such as azidothymidine (AZT) works by blocking: A. Entry of the HIV virus to the circulating volume B. An enzyme needed for viral replication C. The fusion of the HIV to the human cell D. Lymph node infiltration

B. An enzyme needed for viral replication

The nurse lists the opportunistic fungal disease that threatens patients with HIV, which include: (Select all that apply.) A. Herpes simplex B. Aspergillosis C. Pneumocystis jiroveci D. Tuberculosis E. Oral hairy leukoplakia

B. Aspergillosis C. Pneumocystis jiroveci

Which of the following is indicative of a carpopedal spasm in a patient with hypoparathyroidism? a) Cardiac dysrhythmia b) Hand flexing inward c) Moon face and buffalo hump d) Bulging forehead

B. Carpopedal spasm is evidenced by flexion of the elbows and wrists and extension of the carpophalangeal joints and dorsiflexion of the feet.

To avoid the exposure to bacillary angiomatosis (BA), the nurse advises the patient with HIV to avoid: A. Exposure to mosquito bites B. Cats C. Consuming unwashed fruits D. Large crowds of people

B. Cats

In designing a teaching plan for a patient with AIDS, relative to food preparation precautions, the nurse would include the need to: (Select all that apply.) A. Leave produce unwashed to preserve protective spray. B. Check expiration dates on frozen foods. C. Eat three large, well-balanced meals daily. D. Drink a small glass of red wine before each meal to stimulate the appetite. E. Avoid leftovers.

B. Check expiration dates on frozen foods. E. Avoid leftovers.

Which of the following is a clinical manifestation of diabetes insipidus? a) Weight gain b) Excessive thirst c) Low urine output d) Excessive activities

B. Diabetes insipidus (DI), the most common disorder of the posterior lobe of the pituitary gland, is characterized by a deficiency of antidiuretic hormone (ADH) (vasopressin). Excessive thirst (polydipsia) and large volumes of dilute urine characterize the disorder. Urine output may be as high as 20 L in 24 hours. Thirst is excessive and constant. Activities are limited by the frequent need to drink and void. Weight loss develops.

The nurse explains that the enzyme reverse transcriptase transcribes: A. T4-helper cells to RNA B. HIV RNA to HIV DNA C. DNA to mimic CD4 cells D. T4 cells to HIV virons

B. HIV RNA to HIV DNA

The nurse reminds a group of high school students that although homosexual men are stereotyped as victims of HIV, the disease is also observed in: (Select all that apply.) A. Persons sharing living quarters with an HIV-infected person B. Heterosexual partners of an HIV-infected person C. Newborns of an HIV-infected mother D. Breast-fed infants of HIV-infected mothers E. Health care workers who mishandle infected sharps

B. Heterosexual partners of an HIV-infected person C. Newborns of an HIV-infected mother D. Breast-fed infants of HIV-infected mothers E. Health care workers who mishandle infected sharps

Which of the following may occur in the postoperative period of an adrenalectomy because of sudden withdraw of excessive amounts of catecholamines? a) Hyporeflexia b) Hypoglycemia c) Hyperglycemia d) Hypertension

B. Hypotension and hypoglycemia may occur in the postoperative period because of the sudden withdrawal of excessive amounts of catecholamines.

Which medication is the treatment of choice for patients with hyperthyroidism who become pregnant? a) Methimazole (MMI) b) Propylthiouracil (PTU) c) Potassium iodide d) Supersaturated potassium iodide (SSKI)

B. PTU is recommended during the first trimester of pregnancy rather than MMI due to the teratogenic effects of MMI. Due to the risk of hepatotoxicity, PTU should be discontinued after the first trimester and the patient should be switched to MMI for the remainder of the pregnancy and when nursing.

Kaposi's sarcoma is a common problem in patients with AIDS. The nurse should instruct the patient who is diagnosed with AIDS to report which of the following signs of Kaposi's sarcoma? A. Watery diarrhea B. Reddish-purple skin lesions C. Blood-tinged sputum D. Open, bleeding skin lesions

B. Reddish-purple skin lesions

The nurse is completing discharge teaching with a patient with hyperthyroidism who has been treated with radioactive iodine (RAI) at an outpatient clinic. The nurse instructs the patient to do which of the following? a) Discontinue all antithyroid medications. b) Monitor for symptoms of hypothyroidism. c) Continue radioactive precautions with all body secretions. d) Watch for symptoms of hyperthyroidism to disappear within 1 week.

B. Symptoms of hyperthyroidism may later be followed by those of hypothyroidism and myxedema. Hypothyroidism also commonly occurs in patients with previous hyperthyroidism who have been treated with radioiodine or antithyroid medications or thyroidectomy (surgical removal of all or part of the thyroid gland).

A patient with a traumatic brain injury is producing an abnormally large volume of dilute urine. Which alteration to a hormone secreted by the posterior pituitary would the nurse expect to find? a) An increase in antidiuretic hormone b) A deficient production of vasopressin c) A deficient amount of somatostatin d) An increase in oxytocin

B. The most common disorder related to posterior lobe dysfunction is diabetes insipidus, a condition in which abnormally large volumes of dilute urine are excreted as a result of deficient production of vasopressin. Diabetes insipidus may occur following surgical treatment of a brain tumor, secondary to nonsurgical brain tumors, and traumatic brain injury.

Which of the following disorders is characterized by a group of symptoms produced by an excess of free circulating cortisol from the adrenal cortex? a) Hashimoto's disease b) Cushing syndrome c) Addison's disease d) Graves' disease

B. The patient with Cushing syndrome demonstrates truncal obesity, moon face, acne, abdominal striae, and hypertension. Regardless of the cause, the normal feedback mechanisms that control the function of the adrenal cortex become ineffective, and the usual diurnal pattern of cortisol is lost. The signs and symptoms of Cushing syndrome are primarily a result of the oversecretion of glucocorticoids and androgens, although mineralocorticoid secretion also may be affected.

A nursing diagnosis for a patient with HIV/AIDS is Acute Confusion related to disease induced A. anemia B. dementia C. anxiety D. brain damage

B. dementia

Most children with AIDS contracted their disease by the following, except A. from receiving transfusions contaminated with the HIV virus. B. from other children who have AIDS. C. during intrauterine life with HIV-positive mother. D. during the birth process of HIV-positive mother.

B. from other children who have AIDS.

Clinical manifestations - Type I diabetes

Because the onset of Type I DM is rapid, the initial manifestations are usually acute. The osmotic effect of glucose produces polydipsia and polyuria. Polyphagia is a consequence of cellular malnourishment when insulin deficiency prevents use of glucose for energy. Weight loss, weakness, and fatigue may also occur.

A client who was diagnosed with type 1 diabetes 14 years ago is admitted to the medical-surgical unit with abdominal pain. On admission, the client's blood glucose level is 470 mg/dl. Which finding is most likely to accompany this blood glucose level? A. Slow, shallow respirations B. Cool, moist skin C. Rapid, thready pulse D. Arm and leg trembling

C

A client with status asthmaticus requires endotracheal intubation and mechanical ventilation. Twenty-four hours after intubation, the client is started on the insulin infusion protocol. The nurse must monitor the client's blood glucose levels hourly and watch for which early signs and symptoms associated with hypoglycemia? A Dry skin, bradycardia, and somnolence B. Bradycardia, thirst, and anxiety C.Sweating, tremors, and tachycardia D. Polyuria, polydipsia, and polyphagia

C

A nurse is assigned to care for a postoperative client with diabetes mellitus. During the assessment interview, the client reports that he's impotent and says he's concerned about the effect on his marriage. In planning this client's care, the most appropriate intervention would be to: A. provide support for the spouse or significant other. B.encourage the client to ask questions about personal sexuality. C. suggest referral to a sex counselor or other appropriate professional. D. provide time for privacy.

C

A nurse is preparing to administer two types of insulin to a client with diabetes mellitus. What is the correct procedure for preparing this medication? A. The intermediate-acting insulin is withdrawn before the short-acting insulin. B If administered immediately, there is no requirement for withdrawing one type of insulin before another. C The short-acting insulin is withdrawn before the intermediate-acting insulin. D Different types of insulin are not to be mixed in the same syringe.

C

A child is brought into the emergency department with vomiting, drowsiness, and blowing respirations. The father reports that the symptoms have been progressing throughout the day. The nurse suspects diabetic ketoacidosis (DKA). Which action should the nurse take first in the management of DKA? A. Give prescribed antiemetics. B. Administer bicarbonate to correct acidosis. C. Begin fluid replacements. D. Administer prescribed dose of insulin.

C Management of DKA is aimed at correcting dehydration, electrolyte loss, and acidosis before correcting the hyperglycemia with insulin

A client has just returned from a right radical mastectomy. Which action by the unlicensed assistive personnel (UAP) would the nurse consider unsafe? a. Checking the amount of urine in the urine catheter collection bag b. Elevating the right arm on a pillow c. Taking the blood pressure on the right arm d. Encouraging the client to squeeze a rolled washcloth

C Health care professionals need to avoid the arm on the side of the surgery for blood pressure measurement, injections, or blood draws. Since lymph nodes are removed, lymph drainage would be compromised. The pressure from the blood pressure cuff could promote swelling. Infection could occur with injections and blood draws. Checking urine output, elevation of the affected arm on a pillow, and encouraging beginning exercises are all safe postoperative interventions.

A client with bladder cancer is being treated with intravesical administration of immunotherapy. During the treatment, the nurse should do which of the following? a. maintain the client in the trendelenburg position for at least 2 hours b. tell the client to get up to urinate when he feels the urge c. assist the client with changing positions every 15 minutes for 2 hours d. have a gown and mask ready for removal of the fluid from the bladder

C Intravesical therapy is a procedure where Bacillus Calmetter-Guerin (BCG) or interferon-alpha, is instilled directly into the bladder through a catheter. This local administration of therapy reduces the side effects often present with traditional systemic chemotherapy. The goal of intravesical therapy is to prevent bladder tumors from growing or progressing. BCG, a bacterium similar to that sometimes used to vaccinate people against tuberculosis, is a type of intravesical immunotherapy that is very useful for treating recurrent bladder cancer. The medication remains in the bladder for up to 2 hours. During that time, the nurse assists the client in changing position every 15 min to evenly distribute the medication on the bladder and tumor surfaces.

Which finding in a female client by the nurse would receive the highest priority of further diagnostics? a. Tender moveable masses throughout the breast tissue b. A 3-cm firm, defined mobile mass in the lower quadrant of the breast c. Nontender immobile mass in the upper outer quadrant of the breast d. Small, painful mass under warm reddened skin

C Malignant lesions are hard, nontender, and usually located in the upper outer quadrant of the breast and would be the priority for further diagnostic study. The other lesions are benign breast disorders. The tender moveable masses throughout the breast tissue could be a fibrocystic breast condition. A firm, defined mobile mass in the lower quadrant of the breast is a fibroadenoma, and a painful mass under warm reddened skin could be a local abscess or ductal ectasia.

With a history of breast cancer in the family, a 48-year-old female client is interested in learning about the modifiable risk factors for breast cancer. After the nurse explains this information, which statement made by the client indicates that more teaching is needed? a. I am fortunate that I breast-fed each of my three children for 12 months. b. It looks as though I need to start working out at the gym more often. c. I am glad that we can still have wine with every evening meal. d. When I have menopausal symptoms, I must avoid hormone replacement therapy.

C Modifiable risk factors can help prevent breast cancer. The client should lessen alcohol intake and not have wine 7 days a week. Breast-feeding, regular exercise, and avoiding hormone replacement are also strategies for breast cancer prevention.

What comfort measure can only be performed by a nurse, as opposed to an unlicensed assistive personnel (UAP), for a client who returned from a left modified radical mastectomy 4 hours ago? a. Placing the head of bed at 30 degrees b. Elevating the left arm on a pillow c. Administering morphine for pain at a 4 on a 0-to-10 scale d. Supporting the left arm while initially ambulating the client

C Only the nurse is authorized to administer medications, but the UAP could inform the nurse about the rating of pain by the client. The UAP could position the bed to 30 degrees and elevate the clients arm on a pillow to facilitate lymphatic fluid drainage return. The clients arm should be supported while walking at first but then allowed to hang straight by the side. The UAP could support the arm while walking the client.

A client is receiving rituximab (Rituxan) and asks how it works. What response by the nurse is best? a. It causes rapid lysis of the cancer cell membranes. b. It destroys the enzymes needed to create cancer cells. c. It prevents the start of cell division in the cancer cells. d. It sensitizes certain cancer cells to chemotherapy.

C Rituxan prevents the initiation of cancer cell division. The other statements are not accurate.

A 25-year-old client has recently been diagnosed with testicular cancer and is scheduled for radiation therapy. Which intervention by the nurse is best? a. Ask the client about his support system of friends and relatives. b. Encourage the client to verbalize his fears about sexual performance. c. Explore with the client the possibility of sperm collection. d. Provide privacy to allow time for reflection about the treatment.

C Sperm collection is a viable option for a client diagnosed with testicular cancer and should be completed before radiation therapy, chemotherapy, or radical lymph node dissection. The other options would promote psychosocial support but are not the priority intervention.

A 35-year-old woman is diagnosed with stage III breast cancer. She seems to be extremely anxious. What action by the nurse is best? a. Encourage the client to search the Internet for information tonight. b. Ask the client if sexuality has been a problem with her partner. c. Explore the idea of a referral to a breast cancer support group. d. Assess whether there has been any mental illness in her past.

C Support for the diagnosis would be best with a referral to a breast cancer support group. The Internet may be a good source of information, but the day of diagnosis would be too soon. The nurse could assess the frequency and satisfaction of sexual relations but should not assume that there is a problem in that area. Assessment of mental illness is not an appropriate action.

Four clients are receiving tyrosine kinase inhibitors (TKIs). Which of these four clients should the nurse assess first? a. Client with dry, itchy, peeling skin b. Client with a serum calcium of 9.2 mg/dL c. Client with a serum potassium of 2.8 mEq/L d. Client with a weight gain of 0.5 pound (1.1 kg) in 1 day

C TKIs can cause electrolyte imbalances. This potassium level is very low, so the nurse should assess this client first. Dry, itchy, peeling skin can be a problem in clients receiving biologic response modifiers, and the nurse should assess that client next because of the potential for discomfort and infection. This calcium level is normal. TKIs can also cause weight gain, but the client with the low potassium level is more critical.

A client is starting hormonal therapy with tamoxifen (Nolvadex) to lower the risk for breast cancer. What information needs to be explained by the nurse regarding the action of this drug? a. It blocks the release of luteinizing hormone. b. It interferes with cancer cell division. c. It selectively blocks estrogen in the breast. d. It inhibits DNA synthesis in rapidly dividing cells.

C Tamoxifen (Nolvadex) reduces the estrogen available to breast tumors to stop or prevent growth. This drug does not block the release of luteinizing hormone to prevent the ovaries from producing estrogen; leuprolide (Lupron) does this. Chemotherapy agents such as ixabepilone (Ixempra) interfere with cancer cell division, and doxorubicin (Adriamycin) inhibits DNA synthesis in susceptible cells.

The nurse is examining a womans breast and notes multiple small mobile lumps. Which question would be the most appropriate for the nurse to ask? a. When was your last mammogram at the clinic? b. How many cans of caffeinated soda do you drink in a day? c. Do the small lumps seem to change with your menstrual period? d. Do you have a first-degree relative who has breast cancer?

C The most appropriate question would be one that relates to benign lesions that usually change in response to hormonal changes within a menstrual cycle. Reduction of caffeine in the diet has been shown to give relief in fibrocystic breast conditions, but research has not found that it has a significant impact. Questions related to the clients last mammogram or breast cancer history are not related to the nurses assessment.

A 55-year-old African-American client is having a visit with his health care provider. What test should the nurse discuss with the client as an option to screen for prostate cancer, even though screening is not routinely recommended? a. Complete blood count b. Culture and sensitivity c. Prostate-specific antigen d. Cystoscopy

C The prostate-specific antigen test should be discussed as an option for prostate cancer screening. A complete blood count and culture and sensitivity laboratory test will be ordered if infection is suspected. A cystoscopy would be performed to assess the effect of a bladder neck obstruction.

A nurse is caring for a client who has just begun chemotherapy for acute lymphoma. The plan of care indicates that signs of tumor lysis syndrome should be part of the nursing assessment. Which of the following assessment findings are consistent with tumor lysis syndrome? a. polyuria b. muscle weakness c. flank pain d. hypotension

C Tumor lysis syndrome is the development of electrolyte and metabolic disturbances that may result in life-threatening complications if not managed appropriately. Tumor lysis syndrome is caused by the sudden rapid death of cells, particularly cancer cells in clients with leukemia or lymphoma, in response to chemotherapy. Lab results will show high potassium, uric acid, and phosphorous levels and low calcium levels in the blood. If untreated, progression of tumor lysis syndrome may cause acute kidney failure, cardiac arrythmias, seizures, loss of muscle control, and death. Flank pain is a symptom of tumor lysis syndrome secondary to the strain placed on the kidneys to excrete large amounts of intracellular metabolites.

Which instruction should a nurse give to a client with diabetes mellitus when teaching about "sick day rules"? A. "Don't take your insulin or oral antidiabetic agent if you don't eat." B. "It's okay for your blood glucose to go above 300 mg/dl while you're sick." C. "Test your blood glucose every 4 hours." D. D"Follow your regular meal plan, even if you're nauseous."

C The nurse should instruct a client with diabetes mellitus to check his blood glucose levels every 3 to 4 hours and take insulin or an oral antidiabetic agent as usual, even when he's sick. If the client's blood glucose level rises above 300 mg/dl, he should call his physician immediately. If the client is unable to follow the regular meal plan because of nausea, he should substitute soft foods, such as gelatin, soup, and custard

A nurse anticipates that a client who is receiving antineoplastic medications may manifest which of the following side effects? a. gingival hyperplasia b. hirsutism c. aplastic anemia d. weight gain

C Aplastic anemia is a sign of bone marrow suppression, which is a common side effect chemotherapeutic agents. Aplastic anemia results in pancytopenia, a decrease in white blood cells, red blood cells, and platelet count.

A nurse is preparing to discharge a client who had a right radical mastectomy 2 days ago and will be going home with drains still in the incision. When developing a discharge plan of care for this client, which of the following is appropriate for the nurse to include? a. empty the reservoir bulb attached to the drain once a day b. avoid exercises that raise the right arm over the head for 6 weeks c. take your blood pressure on the left arm d. report numbness of the inner right arm to the provider

C Due to the risk of lymphedema from lymph node removal, the client's blood pressure should never be taken in the right arm

A nurse is developing a care plan for a client immediately following a bone marrow transplant. Which of the following post transplact concerns should be the nurse's highest priority? a. pain b. social isolation c. risk for infection d. graft-versus-host disease

C The major cause of death in the first week to 10 days following a bone marrow transplant is infection. A great deal of the client's care should be directed at maintaining a micro-organism-free environment making this the priority concern.

The young man at the HIV clinic tells the nurse how relieved he is that he does not have HIV, because he now has no symptoms at all when just a few weeks ago he felt awful. The nurse's most helpful response would be: A. "A high antibody count can overwhelm HIV infection in the early stage." B. "Antiretroviral drugs are very effective in the first stage in reducing symptoms." C. "In the latent stage the physical symptoms are reduced, but the HIV is still present in the lymph nodes." D. "Flulike symptoms frequently are misdiagnosed as HIV."

C. "In the latent stage the physical symptoms are reduced, but the HIV is still present in the lymph nodes."

slow

does colorectal cancer grow fast or slow?

The nurse cautions a patient with HIV infection who has been prescribed highly active antiretroviral therapy (HAART) that inconsistent administration of the drug can result in the: A. Rapid increase in the symptoms of AIDS B. Addition of another antiretroviral agent to the protocol C. HIV strain becoming resistant to the drug D. Decrease in antibodies in the circulating volume

C. HIV strain becoming resistant to the drug

The very anxious young man comes to the clinic believing that he may have HIV infection because of his persistent influenza-like symptoms and his risky sexual behavior. The nurse anticipates that a positive blood analysis would show: A. Low levels of CD8 cells B. Low levels of T cells C. High levels of HIV-infected cells D. Low levels of antibodies

C. High levels of HIV-infected cells

Which of the following symptoms of thyroid disease is seen in older adults? a) Weight gain b) Hyperactivity c) Atrial fibrillation d) Restlessness

C. Symptoms of thyroid disease seen in older adults include sinus tachycardia or dysrhythmias, increased pulse pressure, and palpitations. Atrial fibrillation occurs in 15% of older adult patients with new onset hyperthyroidism. These changes may be related to increased sensitivity to catecholamines or to changes in neurotransmitter turnover. Older adults may not experience restlessness or hyperactivity.

A patient with hyperthyroidism is concerned about changes in appearance. How can the nurse convey an understanding of the patient's concern and promote effective coping strategies? a) Refer the patient to professional counseling. b) Suggest that the patient wear cosmetics to cover any changes in appearance. c) Reassure the patient that emotional reactions are a result of the disorder and symptoms can be controlled with effective treatment. d) Encourage the patient to participate in outside activities to boost coping strategies.

C. The patient with hyperthyroidism needs reassurance that the emotional reactions being experienced are a result of the disorder and that with effective treatment those symptoms will be controlled. It is important to use a calm, unhurried approach with the patient. Stressful experiences should be minimized and a quiet uncluttered environment should be maintained. The nurse encourages relaxing activities that will not overstimulate the patient. It is important to balance periods of activity with rest.

If the first ELISA is positive, then: A. The diagnosis of AIDS is confirmed. B. A Western blot test is performed on the same sample. C. The test is repeated in 6 to 8 months. D. Another blood sample must be obtained for testing.

C. The test is repeated in 6 to 8 months.

The typical triad of manifestations seen in a patient diagnosed with pheochromocytoma includes all of the following except which of the following? a) Diaphoresis b) Palpitations c) Hypotension d) Headache

C. The typical triad of symptoms seen in patients diagnosed with pheochromocytoma is headache, diaphoresis, and palpitations.

When caring for the patient with AIDS who has cutaneous Kaposi's sarcoma, the nurse would report signs of: A. fatigue B. nausea C. abdominal pain D. weight loss

C. abdominal pain

A patient states that he feels terrific, but a blood test shows that he is HIV-positive. It is important for the nurse to discuss with him that HIV may remain dormant for several years. During this time, the patient A. experiences minor symptoms only. B. Experiences decreased immunity. C. is contagious. D. is not dangerous to anyone.

C. is contagious.

When the patient with HIV is diagnosed with progressive multifocal leukoencephalopathy (PML), the nurse encourages the patient to: A.Take daily exercise for 30 minutes. B.Remove all potted plants from inside the home. C. advanced directives. D. Avoid excessive fats in the diet.

C. prepare advanced directives.

A nurse is reviewing the health record of a client who has syndrome of inappropriate antidiuretic hormone (SIADH). Which of the following laboratory findings should the nurse anticipate? (Select all that apply.) A. Low serum sodium B. High serum potassium C. Decreased urine osmolality D. High urine sodium E. Increased urine-specific gravity

CORRECT: SIADH results in water retention, causing a low serum sodium level. INCORRECT: SIADH does not affect serum potassium levels. INCORRECT: SIADH results in a decrease in urine osmolality. CORRECT: SIADH results in water retention, causing a high urine sodium level. CORRECT: SIADH results in water retention, causing an increase in urine specific gravity

A hypophysectomy is the treatment of choice for which endocrine disorder? a) Hyperthyroidism b) Pheochromocytoma c) Acromegaly d) Cushing syndrome

D. A hypophysectomy is the treatment of choice for the patient diagnosed with Cushing syndrome resulting from excessive production of adrenocorticotropic hormone (ACTH) by a tumor of the pituitary gland.

An adolescent female client who had a successful bone marrow transplant is being discharged. Which of the following information should the nurse include as a part of the discharge plan? Select all that apply a. take you temperature twice a day b. you may return to school if you feel strong enough c. it is important to always wear shoes d. clean your toothbrush weekly with isopropyl alcohol e. avoid using tampons

Clients post bone marrow transplant are immunosuppressed and should continually monitor for signs of infection. A temperature that is greater than 38 C should be reported immediately to the provider A client who had a bone marrow transplant is immunosuppressed and should wear shoes to prevent injury and increase the risk for infection The use of tampons is discouraged because they can disrupt the muscousal layer of the vagina and if left in too long can support the growth of bacteria

A patient with a diagnosis of type 2 diabetes has been vigilant about glycemic control since being diagnosed and has committed to increasing her knowledge about the disease. To reduce her risk of developing diabetic nephropathy in the future, this patient should combine glycemic control with what other preventative measure? A. Maintenance of a low-sodium, low-protein diet B.Subcutaneous injection of 5,000 units of heparin twice daily C. Vigorous physical activity at least three times weekly D Maintenance of healthy blood pressure and prompt treatment of hypertension

D

A type 2 diabetic is ordered metformin (Glucophage) as part of the management regime. Which is the best nursing explanation for the action of this drug in controlling glucose levels? A. Stimulates insulin release B. Reduces the production of glucose by the liver C. Delays digestion of carbohydrates D. Helps tissues use insulin more efficiently

D

After being sick for 3 days, a client with a history of diabetes mellitus is admitted to the hospital with diabetic ketoacidosis (DKA). The nurse should evaluate which diagnostic test results to prevent dysrhythmias? A. Serum chloride level B. Serum sodium level C. Serum calcium level D. Serum potassium level

D

Insulin is secreted by which of the following types of cells? A. Basal cells B. Neural cells C. Melanocytes D. Beta cells

D

The nurse is describing the action of insulin in the body to a client newly diagnosed with type 1 diabetes. Which of the following would the nurse explain as being the primary action? A. It stimulates the pancreatic beta cells. B It aids in the process of gluconeogenesis. C It decreases the intestinal absorption of glucose. D It carries glucose into body cells.

D

When administering insulin to a client with type 1 diabetes, which of the following would be most important for the nurse to keep in mind? A. Technique for injecting B. Duration of the insulin C. Area for insulin injection D. Accuracy of the dosage

D

A hospital patient has been ordered a sliding scale of Humulin R for the duration of her admission. The patient's medication administration record specifies the first administration time of the day at 08:00 and the nurse knows that breakfast trays typically arrive on the unit between 07:45 and 07:50. When should the nurse administer the patient's insulin? A. 08:00 B. 07:45 C. 08:15 D. 07:30

D Short-acting insulin, called regular insulin and marked "R" on the vial, is an unmodified clear solution that usually is administered 20 to 30 minutes before a meal

What is the duration of regular insulin? A. 24 hours B. 3 to 5 hours C. 12 to 16 hours D. 4 to 6 hours

D The duration of regular insulin is 4 to 6 hours; 3 to 5 hours is the duration for rapid-acting insulin such as Novolog. The duration of NPH insulin is 12 to 16 hours. The duration of Lantus insulin is 24 hours

A nurse is assessing a female client who is taking progestins. What assessment finding requires the nurse to notify the provider immediately? a. Irregular menses b. Edema in the lower extremities c. Ongoing breast tenderness d. Red, warm, swollen calf

D All clients receiving progestin therapy are at risk for thromboembolism. A red, warm, swollen calf is a manifestation of deep vein thrombosis and should be reported to the provider. Irregular menses, edema in the lower extremities, and breast tenderness are common side effects of the therapy.

A nurse works with clients who have alopecia from chemotherapy. What action by the nurse takes priority? a. Helping clients adjust to their appearance b. Reassuring clients that this change is temporary c. Referring clients to a reputable wig shop d. Teaching measures to prevent scalp injury

D All of the actions are appropriate for clients with alopecia. However, the priority is client safety, so the nurse should first teach ways to prevent scalp injury.

A client is diagnosed with a fibrocystic breast condition while in the hospital and is experiencing breast discomfort. What comfort measure would the nurse delegate to the unlicensed assistive personnel (UAP)? a. Aid in the draining of the cysts by needle aspiration. b. Teach the client to wear a supportive bra to bed. c. Administer diuretics to decrease breast swelling. d. Obtain a cold pack to temporarily relieve the pain.

D All of the options would be comfort measures for a client with a fibrocystic breast condition. The UAP can obtain the cold or heat therapy. Only the nurse should aid the health care provider with a needle aspiration, teach, and administer medications.

A nurse is caring for a client who has been diagnosed with cancer in situ. The client asks the nurse what type of cancer this is. An appropriate response by the nurse would be that cancer in situ is when abnormal cell production a. has spread to a distant site b. has infiltrated the lymph nodes c. has invaded surrounding tissue d. has developed with a localized area

D Cancer-in-situ is an early stage of cancer that is limited to the site of origin

The nurse has taught a client with cancer ways to prevent infection. What statement by the client indicates that more teaching is needed? a. I should take my temperature daily and when I don't feel well. b. I will wash my toothbrush in the dishwasher once a week. c. I wont let anyone share any of my personal items or dishes. d. Its alright for me to keep my pets and change the litter box.

D Clients should wash their hands after touching their pets and should not empty or scoop the cat litter box. The other statements are appropriate for self-management.

A client with cancer is admitted to a short-term rehabilitation facility. The nurse prepares to administer the clients oral chemotherapy medications. What action by the nurse is most appropriate? a. Crush the medications if the client cannot swallow them. b. Give one medication at a time with a full glass of water. c. No special precautions are needed for these medications. d. Wear personal protective equipment when handling the medications.

D During the administration of oral chemotherapy agents, nurses must take the same precautions that are used when administering IV chemotherapy. This includes using personal protective equipment. These medications cannot be crushed, split, or chewed. Giving one at a time is not needed.

A client with multiple myeloma is admitted to the unit with a white blood cell count of 2,200/mm3. Which of the following foods should the nurse prohibit family members from bringing to the client? a. fried chicken from a fast food shop b. a gift basket of bagels c. a factory-sealed box of chocolates d. a fresh fruit basket

D Raw fruits and vegetables are contraindicated for this client since the skin may harbor bacteria that can cause an infection in this client. These foods should not be brought into the client's room or consumed by the client.

A nurse is developing a plan of care for a client with metastatic lung cancer who is at risk for the development of superior vena cava syndrome. The nurse includes in the plan to assess the client frequently for early manifestations of superior vena cava syndrome including a. irregular cardiac rhythm b. change in the level of consciousness c. wasting of the arms d. facial edema

D Superior vena cava syndrome is a partial occlusion of the superior vena cava. This leads to a lower than normal blood flow through this major vein. Most cases of superior vena cava syndrome are associated with cancers involving the upper chest, such as advanced lung cancers and lymphoma. Cancers that have metastasized to the chest have also been shown to cause superior vena cava syndrome. The earliest manifestations of superior vena cava syndrome are facial and upper extremity edema.

Four weeks after a bone marrow transplant, a client develops a fever and a rash on the hands and feet. Based on these findings the nurse should suspect that the client is experiencing which of the following conditions? a. an allergic response to medication b. side effects of radiation therapy c. veno-occlusive disease of the liver d. graft-versus-host disease

D Temperature elevation and rash on the hands and feet are early manifestations of graft-versus-host disease (GVHD), which usually occurs between 10 and 100 days following the transplant. In GVHD, T-cells from the donor bone marrow attack and destroy vulnerable host cells. Glucocorticoids and cyclosporine may be used to treat the condition.

A nurse and an unlicensed assistive personnel (UAP) are caring for a client with an open radical prostatectomy. Which comfort measure could the nurse delegate to the UAP? a. Administering an antispasmodic for bladder spasms b. Managing pain through patient-controlled analgesia c. Applying ice to a swollen scrotum and penis d. Helping the client transfer from the bed to the chair

D The UAP could aid the client in transferring from the bed to the chair and with ambulation. The nurse would be responsible for medication administration, assessment of swelling, and the application of ice if needed.

A 37-year-old Nigerian woman is at high risk for breast cancer and is considering a prophylactic mastectomy and oophorectomy. What action by the nurse is most appropriate? a. Discourage this surgery since the woman is still of childbearing age. b. Reassure the client that reconstructive surgery is as easy as breast augmentation. c. Inform the client that this surgery removes all mammary tissue and cancer risk. d. Include support people, such as the male partner, in the decision making.

D The cultural aspects of decision making need to be considered. In the Nigerian culture, the man often makes the decisions for care of the female. Women with a high risk for breast cancer can consider prophylactic surgery. If reconstructive surgery is considered, the procedure is more complex and will have more complications compared to a breast augmentation. There is a small risk that breast cancer can still develop in the remaining mammary tissue.

A nurse is teaching a client how to perform a BSE. which finding should the nurse instruct the client to report promptly to her provider? a. lumps that are mobile and tender upon palpation prior to a menstrual period b. multiple round masses that are tender and found in both breasts c. bilaterally darkened areolas d. one non-tender hard lump that is palpated in the upper outer quadrant of the breast

D cancerous tumors are typically hard and non-tender to palpation. The upper-outer quadrant of the breast is the most common site for breast cancer to occur. The client should be taught to notify the provider promptly if a hard, non-tender lump is palpated in the upper-outer quadrant of the breast.

In providing health teaching to a group of clients, it is important for the nurse to emphasize that melanoma is characterized by lesions that have which of the following appearances? a. are predominatly one solid color b. are symmetrical in shape c. are less than 6 mm in diameter d. have an irregular border

D using the ABCD pneumonic a melanotic skin lesion may exhibit one or more of the following characteristics; asymmetry of the shape, border irregularity, color that varies within the lesion, and a diameter than is 6 mm or greater

A 78-year-old patient with type 2 diabetes is brought to the emergency department by his daughter. The patient is found to have a blood glucose level of 623 mg/dL. The patient's daughter reports that the patient recently had a gastrointestinal virus and has been confused for the last 3 hours. A diagnosis of hyperglycemic hyperosmolar nonketotic syndrome (HHNS) is made. What nursing action would be the priority? A. Administering antihypertensive medications B.Administering sodium bicarbonate intravenously for low bicarbonate levels C. Reversing acidosis by administering insulin D. Replacing fluids and electrolytes

D. The overall approach to HHNS includes fluid replacement, correction of electrolyte imbalances, and insulin administration. Insulin administration plays a less important role in the treatment of HHNS because it is not needed for reversal of acidosis, as in diabetic ketoacidosis (DKA). Sodium bicarbonate is not administered to patients with HHNS, as their plasma bicarbonate level is usually normal. Also, antihypertensive medications are not indicated, as hypotension generally accompanies HHNS, due to dehydration

The patient with HIV complains to the home health nurse that he has been having watery diarrhea for the last 10 days. Because the nurse suspects toxoplasmosis, a significant question for the nurse to ask would be: A. "Have you been drinking alcohol?" B. "Have you stopped taking your antiviral medication?" C. "Have you been eating aged cheese or organ meats?" D. "Do you have a cat?"

D. "Do you have a cat?"

The nurse evaluates that teaching, relative to the progression of HIV infections, has been effective when the patient with HIV in the latent stage says: A. "Whew! I thought when I got AIDS that I was a 'goner.'" B. "Now I won't have to take all those expensive drugs that I have been using." C. "I had better get my affairs in order. I don't have a lot of time left." D. "I can still enjoy life and live pretty much as I want for the next several years."

D. "I can still enjoy life and live pretty much as I want for the next several years."

A patient has just been told by his physician that he is HIV-positive. He asks the nurse, "When will I get AIDS?" The nurse's response should be A. "You can expect to develop signs and symptoms of AIDS within 6 months." B. "Most people get AIDS within 3-12 weeks after they are infected with HIV." C. "Don't worry. You may never get AIDS if you eat properly, exercise, and get plenty of rest." D. "It varies with every individual, but the average time is 8-10 years from the time a person is infected, and some go much longer."

D. "It varies with every individual, but the average time is 8-10 years from the time a person is infected, and some go much longer."

A 28-year-old married attorney with one child is in the first trimester of her second pregnancy. A colleague states that she is at no risk for HIV, so she would not need to be counseled about testing for HIV. The nurse's most appropriate response would be A. "We need to review her chart to determine if her first child was infected." B. "She's a professional woman in a monogamous relationship. She obviously is not at risk." C. "Women are not at great risk. The greatest risk is with gay men." D. "The fastest-growing segment of the population with AIDS is women and children. We need to assess her risks."

D. "The fastest-growing segment of the population with AIDS is women and children. We need to assess her risks."

A patient, age 25, has just been admitted to the unit with a diagnosis of AIDS. The nurse's colleague says, "I'm pregnant. It is not safe for me or my baby if I am assigned to his case." The nurse's response should be A. "You should ask for a transfer to another unit because contact with this patient would put you and your baby at risk for AIDS." B. "We should recommend that this patient be transferred to an isolation unit." C. "Wear a mask, gown, and gloves every time you go into his room and use disposable trays, plates, and utensils to serve his meals." D. "This patient would not be a risk for your baby if you use standard precautions and avoid direct contact with blood or body fluids."

D. "This patient would not be a risk for your baby if you use standard precautions and avoid direct contact with blood or body fluids."

The nurse explains that the initial stage of an HIV infection usually lasts from: A. 2 to 4 weeks B. 8 to 12 weeks C. 12 to 16 weeks D. 4 to 8 weeks

D. 4 to 8 weeks

Which of the following diagnostic tests are done to determine a suspected pituitary tumor? a) Radiographs of the abdomen b) Measuring blood hormone levels c) A radioimmunoassay d) A computed tomography (CT) scan

D. A CT or magnetic resonance imaging (MRI) scan is used to diagnose the presence and extent of pituitary tumors.

Which of the following disorders results from excessive secretion of somatotropin (growth hormone)? a) Adrenogenital syndrome b) Cretinism c) Dwarfism d) Acromegaly

D. Acromegaly, an excess of growth hormone (GH) in adults, results in enlargement of peripheral body parts without an increase in height. The patient with acromegaly demonstrates progressive enlargement of peripheral body parts, most commonly the face, head, hands, and feet.

The school nurse uses a chart to demonstrate that, according to statistics from the Centers for Disease Control and Prevention (CDC), the population with the greatest incidence of human immunodeficiency viral (HIV) infection in the United States is: A.Whites B. Latinos C. Asian Americans D. African Americans

D. African Americans

The nurse removes a potted plant from the room of a patient with HIV as a preventive measure against: A. Coccidioidomycosis B. Cytomegalovirus (CMV) C. Candidiasis D. Aspergillosis

D. Aspergillosis

Cardiac effects of hyperthyroidism include which of the following? a) Bradycardia b) Decreased systolic BP c) Decreased pulse pressure d) Palpitations

D. Cardiac effects may include sinus tachycardia, increased pulse pressure, and palpitations. Systolic BP is elevated. The other answers do not apply.

A nurse is completing an assessment of a patient with suspected acromegaly. To assist in making the diagnosis, which of the following questions should the nurse ask? a) "Do you experience skin breakouts?" b) "Is there any family history of acromegaly?" c) "Have you had a recent head injury?" d) "Have you increased your shoe size recently?"

D. Excessive skeletal growth occurs only in the feet, the hands, the superciliary ridge, the molar eminences, the nose, and the chin, giving rise to the clinical picture called acromegaly.

A patient with Cushing syndrome is admitted to the hospital. During the initial assessment, the patient tells the nurse, "The worst thing about this disease is how awful I look. I want to cry every time I look in the mirror." Which of the flowing statements is the best response by the nurse? a) "I do not think you look bad and I am sure your family loves you very much." b) "I can show you how to change your style of dress so that the changes are not so noticeable." c) "I can refer you to a support group. It may help you feel better to talk to someone." d) "If treated successfully, the major physical changes will disappear in time."

D. If treated successfully, the major physical changes associated with Cushing's syndrome disappear in time. The patient may benefit from discussion of the effect the changes have had on his or her self-concept and relationships with others. Weight gain and edema may be modified by a low-carbohydrate, low-sodium diet, and a high protein intake may reduce some of the other bothersome symptoms.

The nurse is aware that the patient has entered the third stage of HIV infection when the patient has: A. Rise in antibody count B. Drop in viral load C. Increase in T4 helper cells D. T-helper CD4 cell count of 500

D. T-helper CD4 cell count of 500

In addition to hand washing, what important precaution must the nurse take when changing the dressing of an AIDS patient? A. Wearing mask B. Strict isolation C. Wearing gloves D. Wearing gown and gloves

D. Wearing gown and gloves

For most people who are HIV-positive, marker antibodies are usually present 10-12 weeks after exposure. The development of these antibodies is called A. opportunistic infection. B. immunocompetence. C. immunodeficiency. D. seroconversion.

D. seroconversion.

While teaching community groups about AIDS, the nurse informs people that the most common method of transmission of the HIV virus currently is: A. perinatal transmission. B. exposure to contaminated blood. C. nonsexual exposure to saliva and tears. D. sexual contact with an HIV-infected partner.

D. sexual contact with an HIV-infected partner.

A 21-year-old male who has been an IV heroin user has been experiencing fever, weight loss, and diarrhea and has been diagnosed as having AIDS. At this time, he has a low-grade fever, severe diarrhea, and a productive cough. He is admitted with Pneumocystis carinii pneumonia. When caring for the patient, the nurse should A. use a gown, mask, and gloves to administer oral medications. B. use a mask when taking the patient's temperature. C. a gown when assisting the patient to use the bedpan. D. use a gown, mask, and gloves when assisting the patient with his bath.

D. use a gown, mask, and gloves when assisting the patient with his bath.

Prediabetes

Defined as impaired glucose intolerance (IGT), impaired fasting glucose (IFG), or both. Individuals diagnosed with pre-diabetes and at increased risk for the development of type 2 diabetes. It is an intermediate stage between normal glucose homeostasis and diabetes where the blood glucose levels are elevated, but not high enough to meet the diagnostic criteria for diabetes.

Polyphagia

Excessive hunger

Polydipsia

Excessive thirst

Drug therapy: Insulin

Exogenous injected insulin is needed when a patient has inadequate insulin to meet specific metabolic needs. People with type I diabetes require exogenous insulin to survive. People with type 2 diabetes, who are usually controlled with diet, exercise and/or OAs, may require exogenous insulin during periods of severe stress, such as illness or surgery. When patient's with type 2 diabetes cannot maintain satisfactory blood glucose levels, exogenous insulin is added to the management plan. Human insulin is derived from common bacteria (E.coli) or yeast cells using recombinant DNA technology. -The insulin regimen that most closely mimics endogenous insulin production is a basal-bolus regimen that uses rapid and short acting (bolus) insulin before meals and long-acting (basal) background insulin once or twice per day. -In addition to mealtime insulin, people with type I diabetes must also use a long- or intermediate-acting (background) insulin to control blood glucose levels between meals and overnight -Combination insulin therapy involves the mixing of short or rapid acting insulin with intermediate acting insulin to provide both mealtime and basal coverage with one injection. Premixed formulas are available for this regimen, but optimal blood glucose control is not as likely because there is less flexibility in dosing.

Type II Diabetes Mellitus - Pathophysiology

FOUR major metabolic abnormalities play a role in the development of type 2 diabetes. 1. INSULIN RESISTANCE = which describes a condition in which body tissues do not respond to the action of insulin because insulin receptors are unresponsive, insufficient in number, or both. Entry of glucose into the cell is impeded, resulting in hyperglycemia. 2. MARKED DECREASE IN THE ABILITY OF THE PANCREAS TO PRODUCE INSULIN = as the B cells become fatigued from the compensatory overproduction of insulin or when B cell mass is lost. 3. INAPPROPRIATE GLUCOSE PRODUCTION BY THE LIVER = instead of properly regulating the release of glucose in response to blood levels, the liver does so in a haphazard way that does not correspond to the body's needs at the time. 4. ALTERATION IN THE PRODUCTION OF HORMONES AND CYTOKINES BY ADIPOSE TISSUE (ADIPOKINES) = adipokines appear to play a role in glucose and fat metabolism and likely contribute to pathophysiology of type 2 diabetes. Individuals with metabolic syndrome are at an increased risk for the development of type 2 diabetes.

Type II Diabetes Mellitus

Formerly known as "adult-onset diabetes" (AODM) or "non-insulin-dependent diabetes" (NIDDM) This is the most prevalent type of diabetes, accounting for greater than 90% of patients with diabetes.

Polyuria

Frequent urination

In reviewing a patient's chart to determine whether she has progressed from HIV disease to AIDS, the nurse should look for A. HIV-positive test result, CD4+ count below 200, history of opportunistic disease. B. Fever, chills, CD4+ count below 200. C. Weight loss, persistent generalized lymphadenopathy, chronic diarrhea. D. CD4+ count below 500, chronic fatigue, night sweats.

HIV postitive test, CD4 count below 200, history of opportunistic infections

Type II Diabetes Mellitus - Pathophysiology

In type II diabetes, the pancreas usually continues to produce some endogenous (self-made) insulin. However, the insulin that is produced either is insufficient for the body's needs or poorly used by the tissues. The presence of endogenous insulin is the major pathophysiologic distinction between type I and type II diabetes. -Risk factors for developing type II diabetes include: -being overweight or obese -being older -having a family hx of type II diabetes -The incidence is increasing in children because of the increasing prevalence of obesity in these individuals.

IFG (impaired fasting glucose)

Is diagnosed when fasting blood glucose levels are 100-125 mg/dL (5.56 to 6.9 mmol/L).

non-small cell carcinoma

Is small-cell or non-small cell carcinoma lung cancer more common?

List the rapid acting insulins

Lispro (Humalog) Aspart (Novalog) Glulisine (Apidra)

Clinical manifestations - Type II diabetes

Manifestations of type 2 diabetes are often non-specific, including fatigue, recurrent infections, prolonged wound healing, and visual changes. Polydipsia, polyuria, and polyphagia may occur.

List of the intermediate acting insulins

NPH (Humulin N, Novolin N)

Intermediate acting insulin

NPH (Humulin N, Novolin N) = cloudy ONSET = 1.5 -4 HRS PEAK = 4-12 HRS DURATION = 12-18 HRS

Prediabetes

People with prediabetes usually do not have symptoms; however, long term damage to the body, especially to the heart and blood vessels, may already be occuring. For this reason, its important to encourage patients to undergo screening and provide teaching about managing risk factors. Those with prediabetes should should have their blood glucose and A1C tested regularly and monitor for symptoms of diabetes, such as polyuria, polyphagia, and polydipsia. Maintaining a healthy weight, exercising regularly, and eating a healthy diet have all been found to reduce the risk of developing overt diabetes in people with prediabetes.

List the short acting insulins

Regular (Humulin R, Novolin R)

Exercise

Regular consistent exercise is an essential part of diabetes and pre-diabetes management. The ADA recommends that people with diabetes perform at least 150 min/week (30 min, 5 days a week) of a moderate-intensity aerobic physical activity. Encourage people with type 2 diabetes to perform resistance training three times a week Exercise decreases insulin resistance and can have a direct effect on lowering blood glucose levels. It also contributes to weight loss, which further decreases insulin resistance, and in addition to it may help reduce TG and LDL cholesterol levels, increase HDL levels, reduce BP, and improve circulation.

Nursing care r/t oral and noninsulin injectable agents

Similar to patients taking insulin. The patient needs to understand the importance of diet and activity plans.

Diagnostic Studies

The diagnosis of diabetes can be made through one of the following four methods. In the absence of unequivocal hyperglycemia, criteria 1 to 3 should be confirmed by repeat testing -A1C of 6.5% -Fasting plasma glucose (FPG) level at or above 126. Fasting is defined as no caloric intake for at least hours -Two hour plasma glucose level at or above 200 during an oral glucose tolerance test, using a glucose load of 75 g. -In a patient with classic symptoms of hyperglycemia (polyuria, polydipsia, unexplained weight loss) or hyperglycemia crisis, a random plasma glucose of at least 200

stage 3 colorectal cancer

The cancer has spread to nearby lymph nodes, but not to other parts of the body.

stage 4 colorectal cancer

The cancer has spread to other parts of the body, such as the liver or lungs.

stage 0 colorectal cancer

The cancer is found only in the innermost lining of the colon or rectum. Referred to as "Carcinoma in situ"

Collaborative Care

The goals of diabetes management are to reduce symptoms, promote well-being, prevent acute complications of hyperglycemia, and prevent or delay the onset and progression of long-term complications. Tools used in the management of diabetes: -Nutrition therapy -Drug therapy -Exercise -Self monitoring of blood glucose The major types of glucose-lowering agents (GLAs) used in the treatment of diabetes are insulin and oral and noninsulin injectable agents. For the majority of people, drug therapy is necessary.

A client receiving chemotherapy every few weeks is told by the provider what is nadir was at his last treatment. When asked by the client what a nadir is, an appropriate answer by the nurse would be which of the following? The nadir is the a. lowest point your blood count reached after treatment b. highest point your blood count reaches after treatment c. point at which the next dose of chemo can be given again d. point at which chemo is determined to be effective

The nadir is the lowest point that blood counts reach prior to recover and after an antineoplastic treatment is given. This occurs approximately 10 days after an antineoplastic treatment

stage 2 colorectal cancer

The tumor extends more deeply into the wall of the colon/rectum. It may have invaded nearby tissue, but has not spread to the lymph nodes.

stage 1 colorectal cancer

The tumor has grown into the inner wall of the colon or rectum but not through it.

Drug therapy: oral and noninsulin injectable agents

This drugs work to improve the mechanisms by which the body produces and uses insulin and glucose. These drugs work on the three defects of type 2 diabetes: insulin resistance, decreased insulin production, and increased hepatic glucose production. Many types of OA's and noninsulin injectable agents are used in the treatment of type 2 diabetes: -BIGUANIDES = primarily reduce glucose production by the liver, but they also enhance insulin sensitivity and improve glucose transport into cells. THE MOST WIDELY USED AGENT IS METFORMIN. -SULFONYLUREAS = increase insulin production by the pancreas -MEGLITINIDES = also increase insulin production from the pancreas, but they are more rapidly absorbed and eliminated than the sulfonyureas, decreasing the potential for hypoglycemia. -ALPHA-GLUCOSIDASE INHIBITORS = slow down the absorption of coarbohydrates in the small intestine. -THIAZOLIDINEDIONES = improve insulin sensitivity, transport, and utilization at target tissues.

Monitoring blood glucose

This helps the patient to make decisions regarding food intake, activity patterns, and medication dosages. Make sure the patient knows how to do this correctly, or else it could mean giving themselves incorrect dosages.

Disease onset in type 2 diabetes

Usually gradual, the person may go years with undetected hyperglycemia that might produce few, if any, symptoms. Many people are on diagnosed on routine testing or when they undergo treatment for other conditions and elevated glucose or glycosylated hemoglobin (A1C) levels are found.

colorectal cancer

a malignant tumor in the lower GI tract

melena

black, tarry, and foul-smelling stools

stage 1 lung cancer

cancer has not spread outside the lung

stage 4 lung cancer

cancer has spread to other organs

stage 2 lung cancer

cancer is found in the lung and nearby lymph nodes

stage 3 lung cancer

cancer is in the lung and lymph nodes in the middle of the chest

Combination therapy (premixed)

combination of short or rapid acting and intermediate = cloudy

stage I

considered cured after surgery

List of the Long acting insulins

degludec (Tresiba) glargine (Lantus) detemir (Levemir)

pleural effusion

excess fluid in the pleural space

Long acting insulin

glargine (Lantus) = clear detemir (Levemir) = clear ONSET = 0.8-4 HRS PEAK = NO PRONOUNCED PEAK DURATION = 24+ HRS

hereditary non-polyposis colorectal cancer

hereditary colon cancer not associated with polyps

abdominoperineal resection

involves the removal of the anus, the rectum, and part of the sigmoid colon along with the associated lymph nodes through incisions made in the abdomen and perineum

Rapid acting insulin

lispro (Humalog) = clear aspart (Novalog) = clear glulisine (Apidra) = clear ONSET = 10-30 MIN PEAK = 30 MIN - 3 HRS DURATION = 3-5 HRS

adenocarcinoma

most colon cancer are....

blood vessels or lymph nodes

most common way for colon cancer to metastasize is...

metastatic disease

most frequently metastasizes to: - lymph nodes - liver - lungs

Familial adenomatous polyposis

mutation that increases the amount of polyps you get, increasing the risk of cancer

Peak of long acting insulin

none

hematochezia

red or maroon-colored stools

Short acting insulin

regular (Humulin R, Novolin R) = clear ONSET = 30 MIN - 1 HR PEAK = 2-5 HRS DURATION = 5-8 HRS

wedge resection

removal of part of the lobe

lobectomy

removal of the entire lobe of the lung

pneumonectomy

removal of the entire lung


Ensembles d'études connexes

Global HRM: Ch. 5 practice questions

View Set

Microbiology Homework Chapter 26 Viruses

View Set

Comparing and Contrasting Two Versions of The War of the Worlds (Quiz)

View Set

Art Appreciation, Study Guide 4 & 5

View Set

CIS Chapter 3 Social Media and Your Digital Identity

View Set

FTC (Federal Trade Commission) VOCAB

View Set